Exam 4: Endocrine (Concepts: Thermoregulation & Glucose Regulation)

Lakukan tugas rumah & ujian kamu dengan baik sekarang menggunakan Quizwiz!

A nurse is caring for a client with a thyrotoxicosis who is at risk for the development of thyroid storm. To detect this complication, the nurse should assess for which sign or symptom? 1. Bradycardia 2. Constipation 3. Hypertension 4. Low-grade temperature

3. Hypertension

The nurse is assessing a client who has a diagnosis of goiter. Which should the nurse expect to note during the assessment of the client? 1. An enlarged thyroid gland 2. The presence of heart damage 3. Client complaints of chronic fatigue 4. Client complaints of slow wound healing

1. An enlarged thyroid gland

A client visits the health care provider's office for a routine physical examination and reports a new onset of intolerance to cold. Since hypothyroidism is suspected, which additional information would be noted during the client's assessment? 1. Weight loss and tachycardia 2. Complaints of weakness and lethargy 3. Diaphoresis and increased hair growth 4. Increased heart rate and respiratory rate

2. Complaints of weakness and lethargy

Which hormone is released from the pancreas? A. Oxytocin B. Prolactin C. Calcitonin D. Somatostatin

D. Somatostatin Somatostatin is a hormone produced by the pancreas that inhibits the release of insulin and glucagon. Oxytocin is a hormone produced by the posterior pituitary gland that acts on the uterus and mammary glands. Prolactin is a hormone produced by the anterior pituitary gland that targets the ovaries and mammary glands in women and testes in men. Calcitonin is a hormone produced by the thyroid gland that interacts with bone tissue.

The nurse is monitoring a client with Graves' disease for signs of thyrotoxic crisis (thyroid storm). Which signs or symptoms, if noted in the client, will alert the nurse to the presence of this crisis? A. Fever and tachycardia B. Pallor and tachycardia C. Agitation and bradycardia D. Restlessness and bradycardia

A. Fever and tachycardia

Which hormonal deficiency would increase the client's risk for fractures? A. Growth hormone B. Follicle-stimulating hormone C. Thyroid-stimulating hormone D. Adrenocorticotropic hormone

A. Growth hormone Growth hormone deficiency causes decrease in bone density, thereby increasing the risk of fractures. Follicle-stimulating hormone deficiency causes amenorrhea, decreased libido, and infertility in women and impotence in men. Thyroid-stimulating hormone deficiency causes menstrual abnormalities and hirsutism. Adrenocorticotropic hormone deficiency causes hypoglycemia and hyponatremia.

What does the nurse state as the etiological factor of nephrogenic diabetes insipidus (DI)? A. Meningitis B. Lithium therapy C. Graves' disease D. Sulfonamide therapy

B. Lithium therapy Lithium therapy is the etiological factor of nephrogenic DI. Central nervous system infections such as meningitis are etiological factors of central DI. Goiter, an enlarged thyroid gland, is commonly seen in clients with Graves' disease. Sulfonamide is a goitrogen that can cause goiter.

A 51-year-old patient with diabetes mellitus is scheduled for a fasting blood glucose level at 8:00 AM. The nurse instructs the patient to only drink water after what time? A. 6:00 PM on the evening before the test B. Midnight before the test C. 4:00 AM on the day of the test D. 7:00 AM on the day of the test

B. Midnight before the test Typically, a patient is ordered to be NPO for 8 hours before a fasting blood glucose level. For this reason, the patient who has a lab draw at 8:00 AM should not have any food or beverages containing any calories after midnight.

In order to ensure a quality specimen and an accurate test result, which instruction should the nurse give a client who is scheduled to undergo urine endocrine testing? A. Start the urine collection when the bladder is full. B. Store the urine specimen in a cooler with ice. C. Store the urine specimen in a home refrigerator. D. Save the urine specimen that begins the collection.

B. Store the urine specimen in a cooler with ice. The urine specimen that is collected for endocrine testing should be stored in a cooler with ice to prevent bacterial growth in the specimen. The nurse should instruct the client to start the urine collection after emptying the bladder. The client should be instructed not to store the urine specimen in a home refrigerator with other food and drinks as it could lead to cross-contamination. The client should be instructed to refrain from saving the urine specimen that begins the collection because the timing for urine collection starts from after the initial voiding specimen.

The nurse is assessing a 75-year-old client for symptoms of hyperglycemia. Which symptom of hyperglycemia is an older adult most likely to exhibit? A. Polyuria B. Polydipsia C. Weight loss D. Infection

D. Infection Signs and symptoms of hyperglycemia in older adults may include fatigue, infection (D), and evidence of neuropathy (e.g., sensory changes). The nurse needs to remember that classic signs and symptoms of hyperglycemia, such as (A, B, and C) and polyphagia, may be absent in older adults.

A patient's recent medical history is indicative of diabetes insipidus. The nurse would perform patient teaching related to which diagnostic test? A. Thyroid scan B. Fasting glucose test C. Oral glucose tolerance D. Water deprivation test

D. Water deprivation test A water deprivation test is used to diagnose the polyuria that accompanies diabetes insipidus. Glucose tests and thyroid tests are not directly related to the diagnosis of diabetes insipidus.

A client's baseline vital signs are as follows: temperature 98.8° F oral, pulse 74 beats/min, respirations 18 breaths/min, and blood pressure 124/76 mm Hg. The client's temperature suddenly spikes to 103° F. Which corresponding respiratory rate should the nurse anticipate in this client as part of the body's response to the change in status? 1. Respiratory rate of 12 breaths/min 2. Respiratory rate of 16 breaths/min 3. Respiratory rate of 18 breaths/min 4. Respiratory rate of 22 breaths/min

4. Respiratory rate of 22 breaths/min

A nurse is monitoring a client receiving glipizide. Which outcome indicates an ineffective response from the medication? A. A decrease in polyuria. B. A decrease in polyphagia. C. A fasting plasma glucose of 100 mg/dL. D. A glycosylated hemoglobin level of 12%.

A glycosylated hemoglobin level of 12%. Glipizide (Glucotrol) is an oral hypoglycemic agent administered to decrease the serum glucose level and the signs and symptoms of hyperglycemia. Therefore, a decrease in both polyuria and polyphagia would indicate a therapeutic response. Laboratory values are also used to monitor a client's response to treatment. A fasting blood glucose level of 100 mg/dL is within normal limits. However, glycosylated hemoglobin of 12% indicates poor glycemic control.

A nurse is formulating a teaching plan for a client recently diagnosed with type 2 diabetes. What interventions should the nurse include to decrease the risk of complications? Select all that apply. A. Examine the feet daily. B. Wear well-fitting shoes. C. Perform regular exercise. D. Powder the feet after showering. E. Visit the primary health care provider weekly. F. Test bathwater with the toes before bathing.

A, B, C. Clients with diabetes often have peripheral neuropathies and are unaware of discomfort or pain in the feet; the feet should be examined every night for signs of trauma. Well-fitting shoes prevent pressure and rubbing that can cause tissue damage and the development of ulcers. Daily exercise increases the uptake of glucose by the muscles and improves insulin use. Powdering the feet after showering may cause a pastelike residue between the toes that may macerate the skin and promote bacterial and fungal growth. Generally, visiting the primary healthcare provider weekly is unnecessary. Clients with diabetes often have peripheral neuropathy and are unable to accurately evaluate the temperature of bathwater, which can result in burns if the water is too hot.

The nurse performs a physical assessment on a 74-year-old woman with possible endocrine dysfunction. The patient's weight was 142 pounds 6 months ago compared to a current weight of 125 pounds. What percent weight change will the nurse document in the patient's health record? A. 12% weight loss B. 17% weight loss C. 25% weight loss D. 74% weight loss

A. 12% weight loss 142 pounds - 125 pounds = 17 pounds; (17/142) × 100 = 12%. Weight change (%) is calculated by dividing the current body weight change by the usual body weight and multiplying the result by 100. Weight change greater than 5% in 1 month, 7.5% in 3 months, or 10% in 6 months is considered severe.

A nurse is reviewing a plan of care for a client with Addison's disease. The nurse notes that the client is at risk for dehydration and suggests nursing interventions that will prevent this occurrence. Which nursing intervention is an appropriate component of the plan of care? Select all that apply. A. Encouraging fluid intake of at least 3L per day B. Encouraging an intake of low-protein foods C. Monitoring for changes in mental status D. Monitoring intake and output E. Maintaining a low-sodium diet

A, C, D. The client at risk for deficient fluid volume should be encouraged to eat regular meals and snacks and to increase the intake of sodium, protein, and complex carbohydrates. Oral replacement of sodium losses is necessary, and maintenance of adequate blood glucose levels is required.

The nurse is reviewing laboratory results for the clinic patients to be seen today. Which patient meets the diagnostic criteria for diabetes mellitus? A. A 48-year-old woman with a hemoglobin A1C of 8.4%. B. A 58-year-old man with a fasting blood glucose of 111 mg/dL C. A 68-year-old woman with a random plasma glucose of 190 mg/dL D. A 78-year-old man with a 2-hour glucose tolerance plasma glucose of 184 mg/dL

A. A 48-year-old woman with a hemoglobin A1C of 8.4% Criteria for a diagnosis of diabetes mellitus include a hemoglobin A1C ≥ 6.5%, fasting plasma glucose level =126 mg/dL, 2-hour plasma glucose level =200 mg/dL during an oral glucose tolerance test, or classic symptoms of hyperglycemia or hyperglycemic crisis with a random plasma glucose =200 mg/dL.

The surgeon was unable to save a patient's parathyroid gland during a radical thyroidectomy. The nurse should consequently pay particular attention to which laboratory value? A. Calcium levels B. Potassium levels C. Blood glucose levels D. Sodium and chloride levels

A. Calcium levels The parathyroid gland plays a key role in maintaining calcium levels. Potassium, sodium, glucose, and chloride are not directly influenced by the loss of the parathyroid gland.

The nurse is assisting a patient with newly diagnosed type 2 diabetes to learn dietary planning as part of the initial management of diabetes. The nurse would encourage the patient to limit intake of which foods to help reduce the percent of fat in the diet? A. Cheese B. Broccoli C. Chicken D. Oranges

A. Cheese Cheese is a product derived from animal sources and is higher in fat and calories than vegetables, fruit, and poultry. Excess fat in the diet is limited to help avoid macrovascular changes.

The nurse has been teaching a patient with diabetes mellitus how to perform self-monitoring of blood glucose (SMBG). During evaluation of the patient's technique, the nurse identifies a need for additional teaching when the patient does what? A. Chooses a puncture site in the center of the finger pad. B. Washes hands with soap and water to cleanse the site to be used. C. Warms the finger before puncturing the finger to obtain a drop of blood. D. Tells the nurse that the result of 110 mg/dL indicates good control of diabetes.

A. Chooses a puncture site in the center of the finger pad. The patient should select a site on the sides of the fingertips, not on the center of the finger pad as this area contains many nerve endings and would be unnecessarily painful. Washing hands, warming the finger, and knowing the results that indicate good control all show understanding of the teaching.

A patient has sought care because of a loss of 25 lb over the past 6 months, during which the patient claims to have made no significant dietary changes. What potential problem should the nurse assess the patient for? A. Thyroid disorders B. Diabetes insipidus C. Pituitary dysfunction D. Parathyroid dysfunction

A. Thyroid disorders Hyperthyroidism is associated with weight loss. Alterations in pituitary function, such as diabetes insipidus, and parathyroid dysfunction are not commonly associated with this phenomenon.

Which statement is true regarding cortisol? A. Cortisol metabolizes free fatty acids. B. Cortisol stimulates gluconeogenesis. C. Cortisol stimulates protein synthesis. D. Cortisol levels decline in stressful situations.

B. Cortisol stimulates gluconeogenesis. Cortisol maintains the blood glucose concentration by stimulating the liver for gluconeogenesis. Gluconeogenesis involves formation of glucose from amino acids and fatty acids. Cortisol mobilizes free fatty acids and inhibits protein synthesis. The blood levels of cortisol increase in stressful conditions.

A nurse is caring for a client with pheochromocytoma. The client asks for a snack and something warm to drink. The appropriate choice for this client to meet nutritional needs would be which of the following? A. Crackers with cheese and tea B. Graham crackers and warm milk C. Toast with peanut butter and cocoa D. Vanilla wafers and coffee with cream and sugar

B. Graham crackers and warm milk The client with pheochromocytoma needs to be provided with a diet that is high in vitamins, minerals, and calories. Of particular importance is that food or beverages that contain caffeine (e.g., chocolate, coffee, tea, and cola) are prohibited.

Which organ has only beta1-receptors? A. Liver B. Heart C. Bladder D. Pancreas

B. Heart The heart has only beta1 receptors, which increase heart rate and contractility. The liver has only alpha receptors. The bladder and pancreas have both alpha and beta receptors.

A nurse is caring for a client after thyroidectomy and monitoring for signs of thyroid storm. The nurse understands that which of the following is a manifestation associated with this disorder? A. Bradycardia B. Hypotension C. Constipation D. Hypothermia

B. Hypotension Clinical manifestations associated with thyroid storm include a fever as high as 106° F (41.1° C), severe tachycardia, profuse diarrhea, extreme vasodilation, hypotension, atrial fibrillation, hyperreflexia, abdominal pain, diarrhea, and dehydration. With this disorder, the client's condition can rapidly progress to coma and cardiovascular collapse.

A nurse working in the diabetes clinic is evaluating a client's success with managing the medical regimen. What is the best indication that a client with type 1 diabetes is successfully managing the disease? A. Reduction in excess body weight B. Stabilization of the serum glucose C. Demonstrated knowledge of the disease D. Adherence to the prescription for insulin

B. Stabilization of the serum glucose A combination of diet, exercise, and medication is necessary to control the disease; the interaction of these therapies is reflected by the serum glucose level. Weight loss may occur with inadequate insulin. Acquisition of knowledge does not guarantee its application. Insulin alone is not enough to control the disease.

In assessing a client diagnosed with primary aldosteronism, the nurse expects the laboratory test results to indicate a decreased serum level of which substance? A. Sodium B. Phosphate C. Potassium D. Glucose

C. Potassium Clients with primary aldosteronism exhibit a profound decline in serum levels of potassium (C); hypokalemia; hypertension is the most prominent and universal sign. The serum sodium level is normal or elevated, depending on the amount of water resorbed with the sodium (A). (B) is influenced by parathyroid hormone (PTH). (D) is not affected by primary aldosteronism.

Growth hormone deficiency results in thinning of bones and increases the risk for pathological fractures. Thyrotropin deficiency results in scalp alopecia and intolerance to cold. Marked increase in the volume of urine output is a sign of diabetes insipidus caused by vasopressin deficiency. A. "I should massage my feet and legs with oil or lotion." B. "I should apply heat intermittently to my feet and legs." C. "I should eat foods high in protein and carbohydrate kilocalories." D. "I should control my blood glucose with diet, exercise, and medication."

D. "I should control my blood glucose with diet, exercise, and medication." Controlling the diabetes decreases the risk of infection; this is the best prevention. Oil or lotion that is not completely absorbed may provide a warm, moist environment for bacterial growth. Coexisting neuropathy may result in injury from heat application. Protein, carbohydrates, and fats must be in an appropriate balance; high carbohydrate intake can provide too many calories.

The nurse should monitor for increases in which laboratory value for the patient as a result of being treated with dexamethasone (Decadron)? A. Sodium B. Calcium C. Potassium D. Blood glucose

D. Blood glucose Hyperglycemia or increased blood glucose level is an adverse effect of corticosteroid therapy. Sodium, calcium, and potassium levels are not directly affected by dexamethasone.

Which clinical manifestation should the nurse expect to note when assessing a client with Addison's disease? A. Edema B. Obesity C. Hirsutism D. Hypotension

D. Hypotension Common manifestations of Addison's disease include postural hypotension from fluid loss, syncope, muscle weakness, anorexia, nausea, vomiting, abdominal cramps, weight loss, depression, and irritability. The manifestations in options 1, 2, and 3 are not associated with Addison's disease.

The patient with an adrenal hyperplasia is returning from surgery for an adrenalectomy. For what immediate postoperative risk should the nurse plan to monitor the patient? A. Vomiting B. Infection C. Thomboembolism D. Rapid BP changes

D. Rapid BP changes The risk of hemorrhage is increased with surgery on the adrenal glands as well as large amounts of hormones being released in the circulation, which may produce hypertension and cause fluid and electrolyte imbalances to occur for the first 24 to 48 hours after surgery. Vomiting, infection, and thromboembolism may occur postoperatively with any surgery.

A 50-year-old female patient smokes, is getting a divorce, and is reporting eye problems. On assessment of this patient, the nurse notes exophthalmos. What other abnormal assessments should the nurse expect to find in this patient? A. Puffy face, decreased sweating, and dry hair B. Muscle aches and pains and slow movements C. Decreased appetite, increased thirst, and pallor D. Systolic hypertension and increased heart rate

D. Systolic hypertension and increased heart rate The patient's manifestations point to Graves' disease or hyperthyroidism, which would also include systolic hypertension and increased heart rate and increased thirst. Puffy face, decreased sweating; dry, coarse hair; muscle aches and pains and slow movements; decreased appetite and pallor are all manifestations of hypothyroidism.

An 18-year-old male patient is undergoing a growth hormone stimulation test. The nurse should monitor the patient for A. hypothermia B. hypertension C. hyperreflexia D. hypoglycemia

D. hypoglycemia Insulin or arginine (agent that stimulates insulin secretion) is administered for a growth hormone stimulation test. The nurse should monitor the patient closely for hypoglycemia. Hypothermia and hypertension are not expected in response to insulin or arginine. Hyperreflexia is an autonomic complication of spinal cord injury.

A nurse teaches a client who has had a thyroidectomy for thyroid cancer to observe for signs of surgically induced hypothyroidism. What should be included in the teaching plan? Select all that apply. A. Dry skin B. Lethargy C. Insomnia D. Tachycardia E. Sensitivity to cold

A, B, E. Dry skin is a response to hypothyroidism that is related to the associated decreased metabolic rate. Lethargy and sensitivity to cold are symptoms related to hypothyroidism that are associated with a decreased metabolic rate. Insomnia and tachycardia are related to hyperthyroidism, not hypothyroidism.

What is the most probable cause for Conn's syndrome in an adult client? A. Genetic cause B. Adrenal adenoma C. High level of angiotensin II D. Elevated level of plasma rennin

B. Adrenal adenoma Conn's syndrome is primary hyperaldosteronism. Excessive secretion of aldosterone by the adrenal glands due to an adrenal adenoma results in Conn's syndrome. Certain types of hyperaldosteronism that are diagnosed in childhood have genetic causes. High levels of angiotensin II that are stimulated by high levels of plasma rennin are a cause for secondary hyperaldosteronism.

The nurse is caring for a child after surgical removal of a brain tumor. The nurse should assess the child for which sign that would indicate that brain stem involvement occurred during the surgical procedure? A. Inability to swallow B. Elevated temperature C. Altered hearing ability D. Orthostatic hypotension

B. Elevated temperature

Which hormone secretion does the nurse state is an example of a positive feedback mechanism? A. Insulin B. Estradiol C. Parathormone D. Catecholamines

B. Estradiol Estradiol secretion pattern is an example of a positive feedback mechanism. Insulin secretion pattern is an example of a negative feedback mechanism. The relationship between calcium and parathormone is also an example of a negative feedback mechanism. Catecholamines secretion is controlled by the nervous system. It is secreted by the sympathetic nervous system.

Which hormones are secreted by the posterior pituitary gland? Select all that apply. A. Oxytocin B. Prolactin C. Corticotropin D. Antidiuretic hormone E. Melanocyte-stimulating hormone

A, D. Oxytocin and antidiuretic hormone (vasopressin) are secreted by the posterior pituitary gland. Prolactin, corticotropin, and melanocyte-stimulating hormones are secreted by the anterior pituitary gland.

Which clinical manifestation is seen in a male client due to deficiency of gonadotropin? A. Decreased fertility B. Increased muscle mass C. Increased bone density D. Decreased urine specific gravity

A. Decreased fertility Deficiency of gonadotropin in males results in clinical manifestation of infertility due to impotence. There is loss of muscle mass and bone density due to gonadotropin deficiency. Clients with diabetes insipidus have decreased urine specific gravity, usually less than 1.005.

A client suspected to have hyperpituitarism is sent by the primary healthcare provider to undergo a suppression test. Which laboratory value would indicate a positive result? A. 3 ng/mL B. 4 ng/mL C. 5 ng/mL D. 6 ng/mL

D. 6 ng/mL When the growth hormone level in a suppression test is above 5 ng/mL, this indicates a positive result, which means the client is suffering from hyperpituitarism. Therefore, 6 ng/mL indicates a positive suppression test. When growth hormone level falls below 5 ng/mL, this indicates a negative result, which means the client is not suffering with hyperpituitarism. Therefore, 3 ng/mL, 4 ng/mL, and 5 ng/mL indicate negative results, and the client does not have hyperpituitarism.

A nurse is assessing a client with Cushing syndrome. Which signs should the nurse expect the client to exhibit? Select all that apply. A. Hirsutism B. Round face C. Pitting edema D. Buffalo hump E. Hypoglycemia

A, B, D. Hirsutism is caused by excess adrenocortical activity associated with Cushing syndrome. A moon face results from an accumulation of adipose tissue associated with hypercortisolism. A buffalo hump results from an accumulation of adipose tissue associated with hypercortisolism. Pitting edema does not occur, except with concurrent severe heart failure. Hypercortisolism increases gluconeogenesis, causing hyperglycemia, not hypoglycemia.

The nurse is developing a plan of care for an older client that addresses interventions to prevent cold discomfort and the development of accidental hypothermia. The nurse should document which desired outcome in the plan of care? A. The client's body temperature is 98° F. B. The client's fingers and toes are cool to touch. C. The client remains in a fetal position when in bed. D. The client complains of coolness in the hands and feet only.

A. The client's body temperature is 98° F.

A client has begun medication therapy with propylthiouracil (PTU). The nurse should assess the client for which condition as an adverse effect of this medication? 1. Joint pain 2. Renal toxicity 3. Hyperglycemia 4. Hypothyroidism

4. Hypothyroidism

Thyroid replacement therapy is prescribed for the client diagnosed with hypothyroidism. The client asks the nurse when the medication will no longer be needed. Which is the appropriate nursing response? 1. "It depends on the results of the laboratory tests." 2. "Most clients require medication for about 1 year." 3. "The medication will need to be continued for life." 4. "You will need to ask your health care provider."

3. "The medication will need to be continued for life."

How does the nurse arrange the events that take place during the promotion of glucose transportation into the cells through cell membranes? 1. Attachment of insulin to receptors. 2. Storage of proinsulin in the pancreas. 3. Secretion of proinsulin by beta cells. 4. Transformation of proinsulin into active insulin.

3. Secretion of proinsulin by beta cells. 2. Storage of proinsulin in the pancreas. 4. Transformation of proinsulin into active insulin. 1. Attachment of insulin to receptors. Proinsulin is a prohormone that is secreted by beta cells and is stored in the beta cells of islets of Langerhans of the pancreas. Active insulin is a protein made up of 51 amino acids; it is produced when C-peptide is removed from the proinsulin. Insulin attaches to receptors present on the target tissues, such as adipose tissue or muscle, where the promotion of glucose transport into the cells through cell membranes occurs.

A client arrives at the clinic complaining of fatigue, lack of energy, constipation, and depression. Following diagnostic studies, hypothyroidism is diagnosed and levothyroxine (Synthroid) is prescribed. The nurse informs the client that which is the expected outcome of the medication? 1. Alleviate depression 2. Increase energy levels 3. Increase blood glucose levels 4. Achieve normal thyroid hormone levels

4. Achieve normal thyroid hormone levels

A client complains of being cold, and the nurse notes the presence of "goose flesh" on the client's arms. The nurse plans care, knowing that which structure is responsible for this response? 1. Arterioles 2. Sweat glands 3. Collagen fibers 4. Arrector pili muscles

4. Arrector pili muscles

Which catecholamine receptor is responsible for increased heart rate? A. Beta 1 receptor B. Beta 2 receptor C. Alpha 1 receptor D. Alpha 2 receptor

A. Beta 1 receptor

Which diagnostic test does the nurse consider to help in identifying the abnormalities of the sella turcica in hyperpituitarism? A. Skull x-ray B. Angiography C. Computer tomography D. Magnetic resonance image

A. Skull x-ray

A nurse is caring for a client following an adrenalectomy and is monitoring for signs of adrenal insufficiency. Which of the following, if noted in the client, indicates signs and symptoms related to adrenal insufficiency? Select all that apply. A. Double vision B. Hypotension C. Mental status changes D. Weakness E. Fever

B, C, D, E. The nurse should be alert to signs and symptoms of adrenal insufficiency in a client following adrenalectomy. These signs and symptoms include weakness, hypotension, fever, and mental status changes. Double vision is generally not associated with this condition.

A nurse is monitoring a client with Graves' disease for signs of thyrotoxic crisis (thyroid storm). Which of the following signs and symptoms, if noted in the client, will alert the nurse to the presence of this crisis? Select all that apply. A. Bradycardia B. Fever C. Sweating D. Agitation E. Pallor

B, C, D. Thyrotoxic crisis (thyroid storm) is an acute, potentially life-threatening state of extreme thyroid activity that represents a breakdown in the body's tolerance to a chronic excess of thyroid hormones. The clinical manifestations include fever greater than 100° F, severe tachycardia, flushing and sweating, and marked agitation and restlessness. Delirium and coma can occur.

The nurse teaches a client with type 2 diabetes nutritional strategies to decrease obesity. Which food item(s) chosen by the client indicates understanding of the teaching? (Select all that apply.) A. White bread B. Salmon C. Broccoli D. Whole milk E. Banana

B, C, E. (B, C, and E) provide fresh fruits, lean meats and fish, vegetables, whole grains, and low-fat dairy products. All are recommended by the American Diabetes Association (ADA) and are a part of the My Plate guidelines recommended by the U.S. Department of Agriculture (USDA). Whole milk (D) is high in fat and is not recommended by ADA. White bread is milled, a process that removes the essential nutrients. It should be avoided for weight loss and is a poor choice for the client with diabetes (A).

What would the nurse anticipate being included in the plan of care for a client who has been diagnosed with Graves' disease? A. Provide a high-fiber diet. B. Provide a restful environment. C. Provide three small meals per day. D. Provide the client with extra blankets.

B. Provide a restful environment. Because of the hypermetabolic state, the client with Graves' disease needs to be provided with an environment that is restful both physically and mentally. Six full meals a day that are well balanced and high in calories are required, because of the accelerated metabolic rate. Foods that increase peristalsis (e.g., high-fiber foods) need to be avoided. These clients suffer from heat intolerance and require a cool environment.

A nurse would expect to note which interventions in the plan of care for a client with hypothyroidism? Select all that apply. A. Provide a cool environment for the client. B. Instruct the client to consume a high-fat diet. C. Instruct the client about thyroid replacement therapy. D. Encourage the client to consume fluids and high-fiber foods. E. Instruct the client to contact the health care provider if episodes of chest pain occur. F. Inform the client that iodine preparations will be prescribed to treat the disorder.

C, D, E. The clinical manifestations of hypothyroidism are the result of decreased metabolism from low levels of thyroid hormone. Interventions are aimed at replacement of the hormones and providing measures to support the signs and symptoms related to a decreased metabolism. The nurse encourages the client to consume a well-balanced diet that is low in fat for weight reduction and high in fluids and high-fiber foods to prevent constipation. The client often has cold intolerance and requires a warm environment. The client would notify the health care provider if chest pain occurs since it could be an indication of overreplacement of thyroid hormone. Iodine preparations are used to treat hyperthyroidism. These medications decrease blood flow through the thyroid gland and reduce the production and release of thyroid hormone.

Which hormones does the nurse state are released by the hypothalamus? Select all that apply. A. FSH B. THS C. MIH D. CRH E. GHRH

C, D, E. The hypothalamus is a small area of nerve and endocrine tissue located beneath the thalamus in the brain. MIH, CRH, and GHRH are released by the hypothalamus. FSH and TSH are released by the anterior pituitary gland.

What are the neurologic manifestations of hyperthyroidism? Select all that apply. A. Fatigue B. Diaphoresis C. Blurred vision D. Exophthalmos E. Shallow respirations

C, D. Blurred vision and exophthalmos are the neurological manifestations of hyperthyroidism. Fatigue is the metabolic manifestation of hyperthyroidism. Diaphoresis, or excessive sweating, is the skin manifestation of hyperthyroidism. Shallow respirations are the cardiopulmonary manifestation of hyperthyroidism.

A nurse observes that a client's urine has a sweet fruity odor. Which information is most important to evaluate when performing a further client assessment? A. Vital signs B. Fluid balance C. Serum glucose level D. Dietary calorie count

C. Serum glucose level Sweet fruity-smelling urine is an indicator of ketoacidosis, which can result from uncontrolled diabetes. Hyperglycemia and hypoglycemia are assessed by serum glucose monitoring. Vital signs, fluid imbalance, and dietary counts have no relation to sweet fruity-smelling urine.

A client with type 1 diabetes mellitus is to begin an exercise program, and the nurse is providing instructions to the client regarding the program. Which of the following should the nurse include in the teaching plan? A. Try to exercise before meal time. B. Administer insulin after exercising. C. Take a blood glucose test before exercising. D. Exercise should be performed during peak times of insulin.

C. Take a blood glucose test before exercising. A blood glucose test performed before exercising provides information to the client regarding the need to eat a snack first. Exercising during the peak times of insulin effect or before mealtime places the client at risk for hypoglycemia. Insulin should be administered as prescribed.

A nurse is assigned to the care of a client hospitalized with a diagnosis of hypothermia. The nurse anticipates that the client will exhibit which findings on assessment of vital signs? A. Increased heart rate and increased blood pressure B. Increased heart rate and decreased blood pressure C. Decreased heart rate and increased blood pressure D. Decreased heart rate and decreased blood pressure

D. Decreased heart rate and decreased blood pressure

A nurse notes in the medical record that a client with Cushing's syndrome is experiencing fluid overload. Which interventions should be included in the plan of care? Select all that apply. A. Monitoring daily weight. B. Monitoring intake and output. C. Maintaining a low-potassium diet. D. Monitoring extremities for edema. E. Maintaining a low-sodium diet.

A, B, D, E. The client with Cushing's syndrome experiencing fluid overload should be maintained on a high-potassium and low-sodium diet. Decreased sodium intake decreases renal retention of sodium and water. Monitoring weight, intake, output, and extremities for edema are all appropriate interventions for such a nursing diagnosis.

While assessing a client during a routine examination, a nurse in the clinic identifies signs and symptoms of hyperthyroidism. Which signs are characteristic of hyperthyroidism? Select all that apply. A. Diaphoresis B. Weight loss C. Constipation D. Protruding eyes E. Cold intolerance

A, B, D. Diaphoresis occurs with hyperthyroidism because of increased metabolism, resulting in hyperthermia. Weight loss occurs with hyperthyroidism because of increased metabolism. Bulging eyes occur with hyperthyroidism and are thought to be related to an autoimmune response of the retroorbital tissue, which causes the eyeballs to enlarge and push forward. Diarrhea occurs because of increased body processes, specifically increased gastrointestinal peristalsis. Heat intolerance occurs because of the increased metabolism associated with hyperthyroidism.

A patient is admitted with diabetes mellitus, malnutrition, and cellulitis. The patient's potassium level is 5.6 mEq/L. The nurse understands that what could be contributing factors for this laboratory result (select all that apply)? A. The level may be increased as a result of dehydration that accompanies hyperglycemia. B. The patient may be excreting extra sodium and retaining potassium because of malnutrition. C. The level is consistent with renal insufficiency that can develop with renal nephropathy. D. The level may be raised as a result of metabolic ketoacidosis caused by hyperglycemia. E. This level demonstrates adequate treatment of the cellulitis and effective serum glucose control.

A, C, D. The additional stress of cellulitis may lead to an increase in the patient's serum glucose levels. Dehydration may cause hemoconcentration, resulting in elevated serum readings. Kidneys may have difficulty excreting potassium if renal insufficiency exists. Finally, the nurse must consider the potential for metabolic ketoacidosis since potassium will leave the cell when hydrogen enters in an attempt to compensate for a low pH. Malnutrition does not cause sodium excretion accompanied by potassium retention. Thus it is not a contributing factor to this patient's potassium level. The elevated potassium level does not demonstrate adequate treatment of cellulitis or effective serum glucose control.

A nurse is reviewing discharge teaching with a client who has Cushing's syndrome. Which statement by the client indicates that the instructions related to dietary management were understood? A. "I can eat foods that contain potassium." B. "I will need to limit the amount of protein in my diet." C. "I am fortunate that I can eat all the salty foods that I enjoy." D. "I am fortunate that I do not need to follow any special diet."

A. "I can eat foods that contain potassium." A diet that is low in calories, carbohydrates, and sodium but ample in protein and potassium content is encouraged for a client with Cushing's syndrome. Such a diet promotes weight loss, the reduction of edema and hypertension, the control of hypokalemia, and the rebuilding of wasted tissue.

When preparing a client for discharge after a thyroidectomy, the nurse teaches the signs of hypothyroidism. When teaching when to call the primary healthcare provider, what statement made by the client shows that teaching was effective? A. "I should call the primary healthcare provider for dry hair and an intolerance to cold." B. "I should call the primary healthcare provider for muscle cramping and sluggishness." C. "I should call the primary healthcare provider for fatigue and an increased pulse rate." D. "I should call the primary healthcare provider for tachycardia and an increase in weight."

A. "I should call the primary healthcare provider for dry hair and an intolerance to cold." Dry, sparse hair and cold intolerance are characteristic responses to low serum thyroxine. Muscle cramping is associated with hypocalcemia. Low thyroxine levels reduce the metabolic rate, resulting in fatigue, but do not increase the pulse rate. Low thyroxine levels reduce the metabolic rate, resulting in weight gain and bradycardia, not tachycardia.

The nurse instructs a 28-year-old man with acromegaly resulting from an unresectable benign pituitary tumor about octreotide (Sandostatin). The nurse should intervene if the patient makes which statement? A. "I will come in to receive this medication IV every 2 to 4 weeks." B. "I will inject the medication in the subcutaneous layer of the skin." C. "The medication will decrease the growth hormone production to normal." D. "If radiation treatment is not effective, I may need to take the medication."

A. "I will come in to receive this medication IV every 2 to 4 weeks." Drugs are most commonly used in patients who have had an inadequate response to or cannot be treated with surgery and/or radiation therapy. The most common drug used for acromegaly is octreotide (Sandostatin), a somatostatin analog that reduces growth hormone levels to within the normal range in many patients. Octreotide is given by subcutaneous injection three times a week. Two long-acting analogs, octreotide (Sandostatin LAR) and lanreotide SR (Somatuline Depot), are available as intramuscular (IM) injections given every 2 to 4 weeks.

The nurse teaches a 38-year-old man who was recently diagnosed with type 1 diabetes mellitus about insulin administration. Which statement by the patient requires an intervention by the nurse? A. "I will discard any insulin bottle that is cloudy in appearance." B. "The best injection site for insulin administration is in my abdomen." C. "I can wash the site with soap and water before insulin administration." D. "I may keep my insulin at room temperature (75o F) for up to a month."

A. "I will discard any insulin bottle that is cloudy in appearance." Intermediate-acting insulin and combination premixed insulin will be cloudy in appearance. Routine hygiene such as washing with soap and rinsing with water is adequate for skin preparation for the patient during self-injections. Insulin vials that the patient is currently using may be left at room temperature for up to 4 weeks unless the room temperature is higher than 86° F (30° C) or below freezing (less than 32° F [0° C]). Rotating sites to different anatomic sites is no longer recommended. Patients should rotate the injection within one particular site, such as the abdomen.

The nurse is assigned to the care of a 64-year-old patient diagnosed with type 2 diabetes. In formulating a teaching plan that encourages the patient to actively participate in management of the diabetes, what should be the nurse's initial intervention? A. Assess patient's perception of what it means to have diabetes. B. Ask the patient to write down current knowledge about diabetes. C. Set goals for the patient to actively participate in managing his diabetes. D. Assume responsibility for all of the patient's care to decrease stress level.

A. Assess patient's perception of what it means to have diabetes. In order for teaching to be effective, the first step is to assess the patient. Teaching can be individualized once the nurse is aware of what a diagnosis of diabetes means to the patient. After the initial assessment, current knowledge can be assessed, and goals should be set with the patient. Assuming responsibility for all of the patient's care will not facilitate the patient's health.

A nurse explains to a client with diabetes that self-monitoring of blood glucose is preferred to urine glucose testing. Why is blood glucose monitoring preferred? A. Blood glucose monitoring is more accurate. B. Blood glucose monitoring is easier to perform. C. Blood glucose monitoring is done by the client. D. Blood glucose monitoring is not influenced by drugs.

A. Blood glucose monitoring is more accurate. Blood glucose testing is a more direct and accurate measure; urine testing provides an indirect measure that can be influenced by kidney function and the amount of time the urine is retained in the bladder. Whereas blood and urine testing is relatively simple, testing the blood involves additional knowledge. Both procedures can be done by the client. Whether or not it is influenced by drugs is not a factor. Although some urine tests are influenced by drugs, there are methods to test urine to bypass this effect.

When the nurse assesses the patient that has pancreatitis, what function may be altered related to the endocrine function of the pancreas? A. Blood glucose regulation B. Increased response to stress C. Fluid and electrolyte regulation D. Regulates metabolic rate of cells

A. Blood glucose regulation The endocrine functions of the pancreas are regulated by α cells that produce and secrete glucagon, β cells that produce and secrete insulin and amylin, delta cells that produce and secrete somatostatin, and F cells that secrete pancreatic polypeptide. Glucagon, insulin, and amylin, and somatostatin all affect blood glucose. Pancreatic polypeptide regulates appetite. Increased response to stress occurs from epinephrine secreted by the adrenal medulla. Fluid and electrolyte regulation occurs in response to several hormones (mineralocorticoids, antidiuretic hormone, parathyroid hormone, calcitonin) from several organs (adrenal cortex, posterior pituitary, parathyroid, thyroid). The metabolic rate of cells is regulated by triiodothyronine (T3) from the thyroid.

A nurse is assisting in preparing a care plan for a client with diabetes mellitus who has hyperglycemia. The nurse focuses on which potential problem for this client? A. Dehydration B. The need for knowledge about the causes of hyperglycemia C. Lack of knowledge about nutrition D. Inability of family to cope with the client's diagnosis

A. Dehydration Increased blood glucose will cause the kidneys to excrete the glucose in the urine. This glucose is accompanied by fluids and electrolytes, causing an osmotic diuresis that leads to dehydration. This fluid loss must be replaced when it becomes severe. Options 2, 3, and 4 may be concerns at some point but are not the priority with hyperglycemia.

The nurse is performing an assessment on a client with a diagnosis of myxedema (hypothyroidism). Which assessment finding should the nurse expect to note in this client? A. Dry skin B. Thin, silky hair C. Bulging eyeballs D. Fine muscle tremors

A. Dry skin

A client reports their lips feel thicker, as well as joint pain and coarse facial features. What should the nurse suspect as the cause of the anterior pituitary hyperfunction? A. Excessive secretion of growth hormone B. Excessive secretion of prolactin hormone C. Excessive secretion of thyroid-stimulating hormone D. Excessive secretion of adrenocorticotropic hormone

A. Excessive secretion of growth hormone Thickened lips, joint pain, and coarse facial features are the symptoms of acromegaly which is caused by pituitary gland hyperfunction leading to excessive secretion of growth hormone. Prolactin hormone hypersecretion can cause hypogonadism, which is loss of sexual characteristics. Thyroid-stimulating hormone hypersecretion can result in increases in both plasma thyroid-stimulating hormone and thyroid hormone levels. Adrenocorticotropic hormone hypersecretion can cause Cushing's disease characterized by increased plasma cortisol levels.

The nurse is evaluating a 45-year-old patient diagnosed with type 2 diabetes mellitus. Which symptom reported by the patient is considered one of the classic clinical manifestations of diabetes? A. Excessive thirst B. Gradual weight gain C. Overwhelming fatigue D. Recurrent blurred vision

A. Excessive thirst The classic symptoms of diabetes are polydipsia (excessive thirst), polyuria, (excessive urine output), and polyphagia (increased hunger). Weight gain, fatigue, and blurred vision may all occur with type 2 diabetes, but are not classic manifestations.

The nurse is caring for a 68-year-old woman after a parathyroidectomy related to hyperparathyroidism. The nurse should administer IV calcium gluconate if the patient exhibits which clinical manifestations? A. Facial muscle spasms or laryngospasms B. Decreased muscle tone or muscle weakness C. Tingling in the hands and around the mouth D. Shortened QT interval on the electrocardiogram

A. Facial muscle spasms or laryngospasms Nursing care for the patient following a parathyroidectomy includes monitoring for a sudden decrease in serum calcium levels causing tetany, a condition of neuromuscular hyperexcitability. If tetany is severe (e.g., muscular spasms or laryngospasms develop), IV calcium gluconate should be administered. Mild tetany, characterized by unpleasant tingling of the hands and around the mouth, may be present but should decrease over time without treatment. Decreased muscle tone, muscle weakness, and shortened QT interval are clinical manifestations of hyperparathyroidism.

The patient has been feeling tired lately and has gained weight; reports thickened, dry skin and increased cold sensitivity even though it is now summer. Which endocrine diagnostic test should be done first? A. Free thyroxine (FT4) B. Serum growth hormone (GH) C. Follicle stimulating hormone (FSH) D. Magnetic resonance imaging (MRI) of the head

A. Free thyroxine (FT4) The manifestations the patient is experiencing could be related to hypothyroidism. Free thyroxine (FT4) is considered a better indicator of thyroid function than total T4 and could be done to evaluate the patient for hypothyroidism. Growth hormone excess could cause thick, leathery, oily skin but does not demonstrate the other manifestations. FSH is manifest with menstrual irregularity and would be useful in distinguishing primary gonadal problems from pituitary insufficiency. MRI is the examination of choice for radiologic evaluation of the pituitary gland and the hypothalamus but would not be the first diagnostic study to further explore the basis of these manifestations.

The patient has been feeling tired lately and has gained weight; reports thickened, dry skin and increased cold sensitivity even though it is now summer. Which endocrine diagnostic test should be done first? A. Free thyroxine (FT4) B. Serum growth hormone (GH) C. Follicle-stimulating hormone (FSH_ D. Magnetic resonance imaging (MRI) of the head

A. Free thyroxine (FT4) The manifestations the patient is experiencing could be related to hypothyroidism. Free thyroxine (FT4) is considered a better indicator of thyroid function than total T4 and could be done to evaluate the patient for hypothyroidism. Growth hormone excess could cause thick, leathery, oily skin but does not demonstrate the other manifestations. FSH is manifest with menstrual irregularity and would be useful in distinguishing primary gonadal problems from pituitary insufficiency. MRI is the examination of choice for radiologic evaluation of the pituitary gland and the hypothalamus but would not be the first diagnostic study to further explore the basis of these manifestations.

Laboratory results have been obtained for a 50-year-old patient with a 15-year history of type 2 diabetes. Which result reflects the expected pattern accompanying macrovascular disease as a complication of diabetes? A. Increased triglyceride levels B. Increased high-density lipoproteins (HDL) C. Decreased low-density lipoproteins (LDL) D. Decreased very-low-density lipoproteins (VLDL)

A. Increased triglyceride levels Macrovascular complications of diabetes include changes to large- and medium-sized blood vessels. They include cerebrovascular, cardiovascular, and peripheral vascular disease. Increased triglyceride levels are associated with these macrovascular changes. Increased HDL, decreased LDL, and decreased VLDL are positive in relation to atherosclerosis development.

A client with type 1 diabetes is transported via ambulance to the emergency department of the hospital. The client has dry, hot, flushed skin and a fruity odor to the breath and is having Kussmaul respirations. Which complication does the nurse suspect that the client is experiencing? A. Ketoacidosis B. Somogyi phenomenon C. Hypoglycemic reaction D. Hyperosmolar nonketotic coma

A. Ketoacidosis Ketoacidosis occurs when insulin is lacking and carbohydrates cannot be used for energy; this increases the breakdown of protein and fat, causing deep, rapid respirations (Kussmaul respirations), decreased alertness, decreased circulatory volume, metabolic acidosis, and an acetone breath. The Somogyi phenomenon is a rebound hyperglycemia induced by severe hypoglycemia; there are not enough data to determine whether this occurred. Hypoglycemia is manifested by cool, moist skin, not hot, dry skin; Kussmaul respirations do not occur with hypoglycemia. Hyperosmolar nonketotic coma usually occurs in clients with type 2 diabetes because available insulin prevents the breakdown of fat.

Based on the clinical manifestations of Cushing's syndrome, which nursing intervention would be appropriate for a client who is newly diagnosed with Cushing's syndrome? A. Monitor glucose levels daily B. Increase intake of fluids high in potassium C. Encourage adequate rest between activities D. Offer the client a sodium-enriched menu

A. Monitor glucose levels daily Cushing's syndrome results from a hypersecretion of glucocorticoids in the adrenal cortex. Clients with Cushing's syndrome often develop diabetes mellitus. Monitoring of serum glucose levels (A) assesses for increased blood glucose levels so that treatment can begin early. A common finding in Cushing's syndrome is generalized edema. Although potassium is needed, it is generally obtained from food intake, not by offering potassium-enhanced fluids (B). Fatigue is usually not an overwhelming factor in Cushing's syndrome, so an emphasis on the need for rest (C) is not indicated A low-calorie, low-carbohydrate, low-sodium diet is not recommended (D).

Which feature in the client indicates hypersecretion of adrenocorticotrophic hormone? A. Moon face B. Lower jaw protrusion C. Heat intolerance D. Barrel-shaped chest

A. Moon face Hypersecretion of adrenocorticotrophic hormone results in Cushing's disease, which is characterized by "moon face" appearance, an abnormal distribution of fat in the face. Protrusion of the lower jaw is a feature of acromegaly, caused by excess secretion of growth hormone. Heat intolerance is seen in clients with excess secretion of thyrotropin. In acromegaly, the client presents with "barrel-shaped" chest appearance.

STUDY MODE: Medical-Surgical Question 57 of 100 ID: 5_54 A 63-year-old client with type 2 diabetes mellitus is admitted for treatment of an ulcer on the heel of the left foot that has not healed with wound care. The nurse observes that the entire left foot is darker in color than the right foot. Which additional symptom should the nurse expect to find? A. Pedal pulses will be weak or absent in the left foot. B. The client will state that the left foot is usually warm. C. Flexion and extension of the left foot will be limited. D. Capillary refill of the client's left toes will be brisk.

A. Pedal pulses will be weak or absent in the left foot. Symptoms associated with decreased blood supply are weak or absent pedal and tibial pulses (A). The client with diabetes experiences vascular scarring as a result of atherosclerotic changes in the peripheral vessels. This results in compromised perfusion to the dependent extremities, which further delays wound healing in the affected foot. Although flexion and extension may be limited (C), depending on the degree of damage, this is not always the case. (B and D) are signs of adequate perfusion of the foot, which would not be expected in this client.

Which gland secretes melatonin? A. Pineal gland B. Thyroid gland C. Adrenal gland D. Parathyroid gland

A. Pineal gland The pineal gland secretes the hormone melatonin, which regulates the circadian rhythm and reproductive system at the onset of puberty. The thyroid gland secretes thyroid hormones. The adrenal gland secretes androgens, corticosteroids, and catecholamines. The thyroid gland secretes the hormone calcitonin.

A nurse is monitoring a client who has been newly diagnosed with diabetes mellitus for signs of complications. Which of the following, if exhibited by client, would indicate hyperglycemia and thus warrant health care provider notification? A. Polyuria B. Bradycardia C. Diaphoresis D. Hypertension

A. Polyuria The classic symptoms of hyperglycemia include polydipsia, polyuria, and polyphagia. Options 2, 3, and 4 are not signs of hyperglycemia.

A nurse is caring for a client with a diagnosis of hypoparathyroidism. The nurse reviews the laboratory results drawn on the client and notes that the calcium level is extremely low. The nurse would expect to note which of the following on data collection of the client? A. Positive Trousseau's sign B. Negative Chvostek's sign C. Unresponsive pupils D. Hyperactive bowel sounds

A. Positive Trousseau's sign Hypoparathyroidism is related to a lack of parathyroid hormone secretion or to a decreased effectiveness of parathyroid hormone on target tissues. The end result of this disorder is hypocalcemia. When serum calcium levels are critically low, the client may exhibit positive Chvostek's and Trousseau's signs, which indicate potential tetany. Options 2, 3, and 4 are not related to the presence of hypocalcemia.

A client with diabetes mellitus visits the health care clinic. The client previously had been well controlled with glyburide (Diabeta), but recently, the fasting blood glucose has been running 180 to 200 mg/dL. Which of the following medications, if added to the client's regimen, may be contributing to the hyperglycemia? A. Prednisone B. Atenolol C. Phenelzine D. Allopurinol

A. Prednisone Prednisone may decrease the effect of oral hypoglycemics, insulin, diuretics, and potassium supplements. Options 2, a β-blocker, and 3, a monoamine oxidase inhibitor, have their own intrinsic hypoglycemic activity. Option 4 decreases urinary excretion of sulfonylurea agents, causing increased levels of the oral medications, which can lead to hypoglycemia.

The nurse is caring for a 56-year-old man receiving high-dose oral corticosteroid therapy to prevent organ rejection after a kidney transplant. What is most important for the nurse to observe related to this medication? A. Signs of infection B. Low blood pressure C. Increased urine output D. Decreased blood glucose

A. Signs of infection Side effects of corticosteroid therapy include increased susceptibility to infection, edema related to sodium and water retention (decreased urine output), hypertension, and hyperglycemia. Other side effects are listed in Table 50-19.

A 24-hour urine test is prescribed for a client who has a tentative diagnosis of pheochromocytoma. What should the nurse do first? A. Start the time of the test after discarding the first voiding. B. Discard the last voiding in the 24-hour time period for the test. C. Insert a urinary retention catheter to promote the collection of urine. D. Strain the urine following each voiding before adding the urine to the container.

A. Start the time of the test after discarding the first voiding. The first voiding is discarded because that urine was in the bladder before the test began and should not be included. The last voiding should be placed in the specimen container because the urine was produced during the 24-hour time frame of the test. Discarding the last void in the 24-hour time period for the test is not necessary; voided specimens are acceptable. Straining the urine following each voiding before adding the urine to the container is not necessary; this is done for clients with renal calculi.

Which adverse effect can be seen in a female client with gonadotropin deficiency and undergoing hormone replacement therapy? A. Thrombosis B. Hypotension C. Dehydration D. Increased thirst

A. Thrombosis A female client with gonadotropin deficiency is treated by replacement therapy of combined hormones, namely estrogen and progesterone. The side effect of this therapy is the increased risk of thrombosis or formation of blood clots in deep veins. Hypertension is a side effect of estrogen-progesterone therapy. Dehydration and increased thirst could indicate vasopressin deficiency.

A patient has sought care because of a loss of 25 lb over the past 6 months, during which the patient claims to have made no significant dietary changes. What potential problem should the nurse assess the patient for? A. Thyroid disorders B. Diabetes insipidus C. Pituitary dysfunction D. Parathyroid dysfuction

A. Thyroid disorders Hyperthyroidism is associated with weight loss. Alterations in pituitary function, such as diabetes insipidus, and parathyroid dysfunction are not commonly associated with this phenomenon.

Which client complaint would alert the nurse to a possible hypoglycemia reaction? A. Tremors B. Anorexia C. Hot, dry skin D. Muscle cramps

A. Tremors Decreased blood glucose levels produce automatic nervous system symptoms, which are classically manifested as nervousness, irritability, and tremors. Option 3 is more likely to occur with hyperglycemia. Options 2 and 4 are unrelated to the signs of hypoglycemia.

A nurse has reinforced dietary instructions to a client with a diagnosis of hypoparathyroidism. The nurse instructs the client to include which of the following items in the diet? A. Vegetables B. Meat C. Fish D. Cereals

A. Vegetables The client with hypoparathyroidism is instructed to follow a calcium-rich diet and to restrict the amount of phosphorus in the diet. The client should limit meat, poultry, fish, eggs, cheese, and cereals. Vegetables are allowed in the diet.

A nurse is caring for a client with pheochromocytoma. The client is scheduled for an adrenalectomy. Durine the preoperative period, the priority nursing action would be to monitor A. Vital signs B. Intake and output C. Blood urea nitrogen (BUN) D. Urine for glucose and acetone

A. Vital signs Hypertension is the hallmark of pheochromocytoma. Severe hypertension can precipitate a brain attack (stroke) or sudden blindness. Although all of the options are accurate nursing interventions for the client with pheochromocytoma, the priority nursing action is to monitor the vital signs, particularly the blood pressure.

The patient with systemic lupus erythematosus had been diagnosed with syndrome of inappropriate antidiuretic hormone (SIADH). What should the nurse expect to include in this patient's plan of care (select all that apply)? A. Obtain weekly weights. B. Limit fluids to 1000 mL per day. C. Monitor for signs of hypernatremia. D. Minimize turning and range of motion. E. Keep the head of the bed at 10 degrees or less elevation.

B, E. The care for the patient with SIADH will include limiting fluids to 1000 mL per day or less to decrease weight, increase osmolality, and improve symptoms; and keeping the head of the bed elevated at 10 degrees or less to enhance venous return to the heart and increase left atrial filling pressure, thereby reducing the release of ADH. The weights should be done daily along with intake and output. Signs of hyponatremia should be monitored, and frequent turning, positioning, and range-of-motion exercises are important to maintain skin integrity and joint mobility.

The nurse is teaching a 60-year-old woman with type 2 diabetes mellitus how to prevent diabetic nephropathy. Which statement made by the patient indicates that teaching has been successful? A. "Smokeless tobacco products decrease the risk of kidney damage." B. "I can help control my blood pressure by avoiding foods high in salt." C. "I should have yearly dilated eye examinations by an ophthalmologist." D. "I will avoid hypoglycemia by keeping my blood sugar above 180 mg/dL."

B. "I can help control my blood pressure by avoiding foods high in salt." Diabetic nephropathy is a microvascular complication associated with damage to the small blood vessels that supply the glomeruli of the kidney. Risk factors for the development of diabetic nephropathy include hypertension, genetic predisposition, smoking, and chronic hyperglycemia. Patients with diabetes are screened for nephropathy annually with a measurement of the albumin-to-creatinine ratio in urine; a serum creatinine is also needed.

The nurse instructs a 22-year-old female patient with diabetes mellitus about a healthy eating plan. Which statement made by the patient indicates that teaching was successful? A. "I plan to lose 25 pounds this year by following a high-protein diet." B. "I may have a hypoglycemic reaction if I drink alcohol on an empty stomach." C. "I should include more fiber in my diet than a person who does not have diabetes." D. "If I use an insulin pump, I will not need to limit the amount of saturated fat in my diet."

B. "I may have a hypoglycemic reaction if I drink alcohol on an empty stomach." The risk for alcohol-induced hypoglycemia is reduced by eating carbohydrates when drinking alcohol. Intensified insulin therapy, such as the use of an insulin pump, allows considerable flexibility in food selection and can be adjusted for alterations from usual eating and exercise habits. However, saturated fat intake should still be limited to less than 7% of total daily calories. Daily fiber intake of 14 g/1000 kcal is recommended for the general population and for patients with diabetes mellitus. High-protein diets are not recommended for weight loss.

The nurse has taught a patient admitted with diabetes, cellulitis, and osteomyelitis about the principles of foot care. The nurse evaluates that the patient understands the principles of foot care if the patient makes what statement? A. "I should only walk barefoot in nice dry weather." B. "I should look at the condition of my feet every day." C. "I am lucky my shoes fit so nice and tight because they give me firm support." D. "When I am allowed up out of bed, I should check the shower water with my toes."

B. "I should look at the condition of my feet every day." Patients with diabetes mellitus need to inspect their feet daily for broken areas that are at risk for infection and delayed wound healing. Properly fitted (not tight) shoes should be worn at all times. Water temperature should be tested with the hands first.

The nurse receives a phone call from a 36-year-old woman taking cyclophosphamide (Cytoxan) for treatment of non-Hodgkin's lymphoma. The patient tells the nurse that she has muscle cramps and weakness and very little urine output. Which response by the nurse is best? A. "Start taking supplemental potassium, calcium, and magnesium." B. "Stop taking the medication now and call your health care provider." C. "These symptoms will decrease with continued use of the medication." D. "Increase fluids to 3000 mL per 24 hours to improve your urine output."

B. "Stop taking the medication now and call your health care provider." Cyclophosphamide may cause syndrome of inappropriate antidiuretic hormone (SIADH). Medications that stimulate the release of ADH should be avoided or discontinued. Treatment may include restriction of fluids to 800 to 1000 mL per day. If a loop diuretic such as furosemide (Lasix) is used to promote diuresis, supplements of potassium, calcium, and magnesium may be needed.

A 54-year-old patient admitted with type 2 diabetes asks the nurse what "type 2" means. What is the most appropriate response by the nurse? A. "With type 2 diabetes, the body of the pancreas becomes inflamed." B. "With type 2 diabetes, insulin secretion is decreased, and insulin resistance is increased." C. "With type 2 diabetes, the patient is totally dependent on an outside source of insulin." D. "With type 2 diabetes, the body produces autoantibodies that destroy β-cells in the pancreas."

B. "With type 2 diabetes, insulin secretion is decreased, and insulin resistance is increased." In type 2 diabetes mellitus, the secretion of insulin by the pancreas is reduced, and/or the cells of the body become resistant to insulin. The pancreas becomes inflamed with pancreatitis. The patient is totally dependent on exogenous insulin and may have had autoantibodies destroy the β-cells in the pancreas with type 1 diabetes mellitus.

Which patient with type 1 diabetes mellitus would be at the highest risk for developing hypoglycemic unawareness? A. A 58-year-old patient with diabetic retinopathy B. A 73-year-old patient who takes propranolol (Inderal) C. A 19-year-old patient who is on the school track team D. A 24-year-old patient with a hemoglobin A1C of 8.9%

B. A 73-year-old patient who takes propranolol (Inderal) Hypoglycemic unawareness is a condition in which a person does not experience the warning signs and symptoms of hypoglycemia until the person becomes incoherent and combative or loses consciousness. Hypoglycemic awareness is related to autonomic neuropathy of diabetes that interferes with the secretion of counterregulatory hormones that produce these symptoms. Older patients and patients who use â-adrenergic blockers (e.g., propranolol) are at risk for hypoglycemic unawareness.

The nursing instructor asks a nursing student to identify the risk factors associated with the development of thyrotoxicosis. The student demonstrates understanding of the risk factors by identifying an increased risk for thyrotoxicosis in which client? A. A client with hypothyroidism. B. A client with Graves' disease who is having surgery. C. A client with diabetes mellitus. D. A client with diabetes mellitus scheduled for debridement of a foot ulcer.

B. A client with Graves' disease who is having surgery.

A nurse is reviewing the postoperative prescriptions for a client who had a transsphenoidal hypophysectomy. Which health care provider's prescription, if noted on the record, indicates the need for clarification? A. Instruct the client about the need for a Medic-Alert bracelet. B. Apply a loose dressing if any clear drainage is noted. C. Monitor vital signs and neurological status. D. Instruct the client to avoid blowing the nose.

B. Apply a loose dressing if any clear drainage is noted. The nurse should observe for clear nasal drainage, constant swallowing, and a severe, persistent, generalized, or frontal headache. These signs and symptoms indicate cerebrospinal fluid leak into the sinuses. If clear drainage is noted following this procedure, the health care provider needs to be notified immediately. Options 1, 3, and 4 indicate appropriate postoperative interventions.

The nurse is caring for a 40-year-old man who has begun taking levothyroxine (Synthroid) for recently diagnosed hypothyroidism. What information reported by the patient is most important for the nurse to further assess? A. Weight gain or weight loss B. Chest pain and palpitations C. Muscle weakness and fatigue D. Decreased appetite and constipation

B. Chest pain and palpitations Levothyroxine (Synthroid) is used to treat hypothyroidism. Any chest pain or heart palpitations or heart rate greater than 100 beats/minute experienced by a patient starting thyroid replacement should be reported immediately, and an electrocardiogram (ECG) and serum cardiac enzyme tests should be performed.

A nurse has a prescription to administer a medication to a client who is experiencing shivering as a result of hyperthermia. Which medication should the nurse anticipate to be prescribed? A. Buspirone (BuSpar) B. Chlorpromazine (Thorazine) C. Prochlorperazine (Compazine) D. Fluphenazine (Prolixin Decanoate)

B. Chlorpromazine (Thorazine)

A client presents with chief complaints of unexplained weight gain and back pain from a compression fracture of the vertebrae. On assessment, there is truncal obesity with excessively thin extremities, a moon-shaped face, a buffalo hump, thin hair, and adult acne. The symptoms described are suggestive of what disease? A. Addison's disease B. Cushing disease C. Multiple sclerosis D. Kaposi sarcoma

B. Cushing disease Common symptoms of Cushing disease are weight gain, truncal obesity, buffalo hump, and moon face because of deposits of adipose tissue. The condition is caused by excess cortisol secretion caused by hypersecretion of adrenocorticotropic hormone (ACTH). Other characteristics are diabetes mellitus, muscle wasting, osteoporosis, ecchymosis, and slow healing of wounds. Addison disease is adrenal insufficiency. Symptoms of Addison disease include hypotension, dehydration, hypoglycemia, and hyperpigmentation of the skin. Multiple sclerosis is a progressive disease involving destruction of the myelin sheath, leading to nerve damage. Kaposi sarcoma is a cancer associated with acquired immunodeficiency syndrome (AIDS).

A client is diagnosed with hyperthyroidism, and surgery is scheduled because the client refuses ablation therapy. While awaiting the surgical date, what instruction should the nurse teach the client? A. Consciously attempt to calm down. B. Eliminate coffee, tea, and cola from the diet. C. Keep the home warm, and use an extra blanket at night. D. Schedule activities during the day to overcome lethargy.

B. Eliminate coffee, tea, and cola from the diet. Coffee, tea, and cola contain caffeine, which may increase thyroid activity. Hyperactivity is a physiological response; it is not under conscious control. The increased metabolic rate associated with hyperthyroidism will make the client feel warm; a cool environment is needed. Hyperactivity is a problem, and the client should be encouraged to rest.

Which clinical manifestation occurs in a client with vasopressin deficiency? A. Impotence B. Hypotension C. Amenorrhea D. Decreased libido

B. Hypotension Vasopressin regulates fluid level and blood pressure. A vasopressin deficiency causes hypotension. Impotence, amenorrhea, and decreased libido in both men and women are clinical manifestations of luteinizing and follicle-stimulating hormone deficiencies.

The patient is brought to the ED following a car accident and is wearing medical identification that says she has Addison's disease. What should the nurse expect to be included in the collaborative care of this patient? A. Low sodium diet B. Increased glucocorticoid replacement C. Suppression of pituitary ACTH synthesis D. Elimination of mineralocorticoid replacement

B. Increased glucocorticoid replacement The patient with Addison's disease needs lifelong glucocorticoid and mineralocorticoid replacement and has an increased need with illness, injury, or stress, as this patient is experiencing. The patient with Addison's may also need a high sodium diet. Suppression of pituitary ACTH synthesis is done for Cushing syndrome. Elimination of mineralocorticoid replacement cannot be done for Addison's disease.

A client with Cushing's disease is being admitted to the hospital after a stab wound to the abdomen. The nurse plans care and places highest priority on which potential problem? A. Nervousness B. Infection C. Concern about appearance D. Inability to care for self

B. Infection The client with a stab wound has a break in the body's first line of defense against infection. The client with Cushing's disease is at great risk for infection because of excess cortisol secretion and subsequent impaired antibody function and decreased proliferation of lymphocytes. The client may also have a potential for the problems listed in the other options but these are not the highest priority at this time.

A nurse is collecting data on a client admitted to the hospital with a diagnosis of myxedema. Which data collection technique will provide data necessary to support the admitting diagnosis? A. Auscultation of lung sounds B. Inspection of facial features C. Percussion of the thyroid gland D. Palpation of the adrenal gland

B. Inspection of facial features Inspection of facial features will reveal the characteristic coarse features, presence of edema around the eyes and face, and a blank expression that are characteristic of myxedema. The techniques in the remaining options will not reveal any data that would support the diagnosis of myxedema.

A nurse is caring for a postoperative parathyroidectomy client. Which of the following would require the nurse's immediate attention? A. Incisional pain B. Laryngeal stridor C. Difficulty voiding D. Abdominal cramping

B. Laryngeal stridor During the postoperative period, the nurse carefully observes the client for signs of hemorrhage, which cause swelling and the compression of adjacent tissue. Laryngeal stridor is a harsh, high-pitched sound heard on inspiration and expiration that is caused by the compression of the trachea and that leads to respiratory distress. It is an acute emergency situation that requires immediate attention to avoid the complete obstruction of the airway.

Which abnormal laboratory finding indicates that a client with diabetes needs further evaluation for diabetic nephropathy? A. Hypokalemia B. Microalbuminuria C. Elevated serum lipid levels D. Ketonuria

B. Microalbuminuria Microalbuminuria (B) is the earliest sign of diabetic nephropathy and indicates the need for follow-up evaluation. Hyperkalemia, not (A), is associated with end-stage renal disease caused by diabetic nephropathy. (C) may be elevated in end-stage renal disease. (D) may signal the onset of diabetic ketoacidosis (DKA).

The nurse is beginning to teach a diabetic patient about vascular complications of diabetes. What information is appropriate for the nurse to include? A. Macroangiopathy does not occur in type 1 diabetes but rather in type 2 diabetics who have severe disease. B. Microangiopathy is specific to diabetes and most commonly affects the capillary membranes of the eyes, kidneys, and skin. C. Renal damage resulting from changes in large- and medium-sized blood vessels can be prevented by careful glucose control. D. Macroangiopathy causes slowed gastric emptying and the sexual impotency experienced by a majority of patients with diabetes.

B. Microangiopathy is specific to diabetes and most commonly affects the capillary membranes of the eyes, kidneys, and skin. Microangiopathy occurs in diabetes mellitus. When it affects the eyes, it is called diabetic retinopathy. When the kidneys are affected, the patient has nephropathy. When the skin is affected, it can lead to diabetic foot ulcers. Macroangiopathy can occur in either type 1 or type 2 diabetes and contributes to cerebrovascular, cardiovascular, and peripheral vascular disease. Sexual impotency and slowed gastric emptying result from microangiopathy and neuropathy.

A patient with diabetes mellitus who has multiple infections every year needs a mitral valve replacement. What is the most important preoperative teaching the nurse should provide to prevent a cardiac infection postoperatively? A. Avoid sick people and wash hands. B. Obtain comprehensive dental care. C. Maintain hemoglobin A1c below 7%. D. Coughing and deep breathing with splinting

B. Obtain comprehensive dental care. A person with diabetes is at high risk for postoperative infections. The most important preoperative teaching to prevent a postoperative infection in the heart is to have the patient obtain comprehensive dental care because the risk of septicemia and infective endocarditis increases with poor dental health. Avoiding sick people, hand washing, maintaining hemoglobin A1c below 7%, and coughing and deep breathing with splinting would be important for any type of surgery, but not the priority with mitral valve replacement for this patient.

Which assessment parameter is of highest priority when caring for a patient undergoing a water deprivation test? A. Serum glucose B. Patient weight C. Arterial blood gases D. Patient temperature

B. Patient weight A patient is at risk for severe dehydration during a water deprivation test. The test should be discontinued and the patient rehydrated if the patient's weight drops more than 2 kg at any time. The other assessment parameters do not assess fluid balance.

After several diagnostic tests, a client is diagnosed with diabetes insipidus. The nurse understands that which symptom is indicative of this disorder? A. Diarrhea B. Polydipsia C. Weight gain D. Blurred vision

B. Polydipsia Polydipsia and polyuria are classic symptoms of diabetes insipidus. The urine is pale in color, and its specific gravity is low. Anorexia and weight loss occur. Diarrhea, weight gain, and blurred vision are not manifestations of the disorder.

The nurse interviews a 50-year-old man with a history of type 2 diabetes mellitus, chronic bronchitis, and osteoarthritis who has a fasting blood glucose of 154 mg/dL. Which medications, if taken by the patient, may raise blood glucose levels? A. Glargine B. Prednisone C. Metformin D. Acetaminophen

B. Prednisone Prednisone is a corticosteroid that may cause glucose intolerance in susceptible patients by increasing gluconeogenesis and insulin resistance. Insulin (e.g., glargine) and metformin (an oral hypoglycemic agent) decrease blood glucose levels. Acetaminophen has a glucose-lowering effect.

The nurse is planning care for a client with diabetes mellitus who has gangrene of the toes to the midfoot. Which goal should be included in this client's plan of care? A. Restore skin integrity. B. Prevent infection. C. Promote healing. D. Improve nutrition.

B. Prevent infection. The prevention of infection is a priority goal for this client (B). Gangrene is the result of necrosis (tissue death). If infection develops, there is insufficient circulation to fight the infection and the infection can result in osteomyelitis or sepsis. Because tissue death has already occurred, (A and C) are unattainable goals. (D) is important but of less priority than (B).

A client treated for an episode of hyperthermia is being discharged to home. The nurse determines that the client needs clarification of discharge instructions if the client states a need to perform which action? A. Increase fluid intake. B. Resume full activity level. C. Stay in a cool environment when possible. D. Monitor voiding for adequacy of urine output.

B. Resume full activity level.

A nurse reinforces teaching with a client with diabetes mellitus regarding differentiating between hypoglycemia and ketoacidosis. The client demonstrates an understanding of the teaching by stating that glucose will be tested if which symptom develops? A. Polyuria B. Shakiness C. Blurred vision D. Fruity breath odor

B. Shakiness Shakiness is a sign of hypoglycemia, and it would indicate the need for food or glucose. Fruity breath odor, blurred vision, and polyuria are signs of hyperglycemia.

A client, visiting the health center, reports feeling nervous, irritable, and extremely tired. The client says to the nurse, "Although I eat a lot of food, I have frequent bouts of diarrhea and am losing weight." The nurse observes a fine hand tremor, an exaggerated reaction to external stimuli, and a wide-eyed expression. What laboratory tests may be prescribed to determine the cause of these signs and symptoms? A. Partial thromboplastin time (PTT) and prothrombin time (PT) B. T3, T4, and thyroid-stimulating hormone (TSH) C. Venereal disease research laboratory (VDRL) test and complete blood count (CBC) D. Adrenocorticotropic hormone (ACTH), antidiuretic hormone (ADH), and corticotropin-releasing factor (CRF)

B. T3, T4, and thyroid-stimulating hormone (TSH) T3, T4, and TSH provide a measure of thyroid hormone production; an increase is associated with the client's signs and symptoms. PT and PTT assess blood coagulation. The VDRL test is for syphilis; the CBC assesses the hematopoietic system. ACTH stimulates the synthesis and secretion of adrenal cortical hormones. ADH increases water reabsorption by the kidney. CRF triggers the release of ACTH.

The nurse is providing discharge instructions to a patient with diabetes insipidus. Which instructions regarding desmopressin acetate (DDAVP) would be most appropriate? A. The patient can expect to experience weight loss resulting from increased diuresis. B. The patient should alternate nostrils during administration to prevent nasal irritation. C. The patient should monitor for symptoms of hypernatremia as a side effect of this drug. D. The patient should report any decrease in urinary elimination to the health care provider.

B. The patient should alternate nostrils during administration to prevent nasal irritation. DDAVP is used to treat diabetes insipidus by replacing the antidiuretic hormone that the patient is lacking. Inhaled DDAVP can cause nasal irritation, headache, nausea, and other signs of hyponatremia. Diuresis will be decreased and is expected, and hypernatremia should not occur.

The nurse is assessing a male client with acute pancreatitis. Which finding requires the most immediate intervention by the nurse? A. The client's amylase level is three times higher than the normal level. B. While the nurse is taking the client's blood pressure, he has a carpal spasm. C. On a 1 to 10 scale, the client tells the nurse that his epigastric pain is at 7. D. The client states that he will continue to drink alcohol after going home.

B. While the nurse is taking the client's blood pressure, he has a carpal spasm. A positive Trousseau sign (B) indicates hypocalcemia and always requires further assessment and intervention, regardless of the cause (40% to 75% of those with acute pancreatitis experience hypocalcemia, which can have serious, systemic effects). A key diagnostic finding of pancreatitis is serum amylase and lipase levels that are two to five times higher than the normal value (A). Severe boring pain is an expected symptom for this diagnosis (C), but dealing with the hypocalcemia is a priority over administering an analgesic. Long-term planning and teaching (D) do not have the same immediate importance as a positive Trousseau sign.

A nurse has reinforced instructions to the client with hyperparathyroidism regarding home care measures related to exercise. Which statement by the client indicates a need for further instruction? Select all that apply. A. "I enjoy exercising but I need to be careful." B. "I need to pace my activities throughout the day." C. "I need to limit playing football to only the weekends." D. "I should gauge my activity level by my energy level." E. "I should exercise in the evening to encourage a good sleep pattern."

C, E. The client should be instructed to avoid high-impact activity or contact sports such as football. Exercising late in the evening may interfere with restful sleep. The client with hyperparathyroidism should pace activities throughout the day and plan for periods of uninterrupted rest. The client should plan for at least 30 minutes of walking each day to support calcium movement into the bones. The client should be instructed to use energy level as a guide to activity.

A nurse provides dietary instructions to a client with diabetes mellitus regarding the prescribed diabetic diet. Which statement, if made by the client, indicates the need for further teaching? A. "I'll eat a balanced meal plan." B. "I need to drink diet soft drinks." C. "I need to buy special diabetic foods." D. "I will snack on fruit instead of cake."

C. "I need to buy special diabetic foods." It is important to emphasize to the client and family that they are not eating a diabetic diet but rather following a balanced meal plan. Adherence to nutrition principles is an important component of diabetic management, and an individualized meal plan should be developed for the client. It is not necessary for the client to purchase special dietetic foods.

The patient received regular insulin 10 units subcutaneously at 8:30 PM for a blood glucose level of 253 mg/dL. The nurse plans to monitor this patient for signs of hypoglycemia at which time related to the insulin's peak action? A. 8:40 PM to 9:00 PM B. 9:00 PM to 11:30 PM C. 10:30 PM to 1:30 AM D. 12:30 AM to 8:30 AM

C. 10:30 PM to 1:30 AM Regular insulin exerts peak action in 2 to 5 hours, making the patient most at risk for hypoglycemia between 10:30 PM and 1:30 AM. Rapid-acting insulin's onset is between 10-30 minutes with peak action and hypoglycemia most likely to occur between 9:00 PM and 11:30 PM. With intermediate acting insulin, hypoglycemia may occur from 12:30 AM to 8:30 AM.

The nurse is providing care to a client being treated for bacterial cystitis. What is the goal before discharge for this client? A. Understand the need to drink 4 L of water per day, an essential measure to prevent dehydration B. Be able to identify dietary restrictions and plan menus C. Achieve relief of symptoms and maintain kidney function D. Recognize signs of bleeding, a complication associated with this type of procedure

C. Achieve relief of symptoms and maintain kidney function Relief of symptoms and continued urine output are measurable responses to therapy and are the desired outcomes. Four liters of water per day is too much fluid; 2 to 3 liters a day is recommended to flush the bladder and urethra. Dietary restrictions are not necessary with cystitis. Bleeding is not a complication associated with this procedure.

A nurse is assisting with preparing a teaching plan for the client with diabetes mellitus regarding proper foot care. Which instruction should be included in the plan of care? A. Soak the feet in hot water. B. Avoid using soap to wash the feet. C. Apply a moisturizing lotion to dry feet, but not between the toes. D. Always have a podiatrist cut your toenails; never cut them yourself.

C. Apply a moisturizing lotion to dry feet, but not between the toes. The client should use a moisturizing lotion on his or her feet, but should avoid applying the lotion between the toes. The client should also be instructed to not soak the feet and to avoid hot water to prevent burns. The client may cut the toenails straight across and even with the toe itself, but he or she should consult a podiatrist if the toenails are thick or hard to cut or if his or her vision is poor. The client should be instructed to wash the feet daily with a mild soap.

A 65-year-old patient with type 2 diabetes has a urinary tract infection (UTI). The unlicensed assistive personnel (UAP) reported to the nurse that the patient's blood glucose is 642 mg/dL and the patient is hard to arouse. When the nurse assesses the urine, there are no ketones present. What collaborative care should the nurse expect for this patient? A. Routine insulin therapy and exercise B. Administer a different antibiotic for the UTI. C. Cardiac monitoring to detect potassium changes D. Administer IV fluids rapidly to correct dehydration.

C. Cardiac monitoring to detect potassium changes This patient has manifestations of hyperosmolar hyperglycemic syndrome (HHS). Cardiac monitoring will be needed because of the changes in the potassium level related to fluid and insulin therapy and the osmotic diuresis from the elevated serum glucose level. Routine insulin would not be enough, and exercise could be dangerous for this patient. Extra insulin will be needed. The type of antibiotic will not affect HHS. There will be a large amount of IV fluid administered, but it will be given slowly because this patient is older and may have cardiac or renal compromise requiring hemodynamic monitoring to avoid fluid overload during fluid replacement.

A nurse is caring for a client with pheochromocytoma. Which data would indicate a potential complication associated with this disorder? A. A urinary output of 50 mL/hr B. A coagulation time of 5 minutes C. Congestion heard on auscultation of the lungs D. A blood urea nitrogen (BUN) level of 20 mg/dL

C. Congestion heard on auscultation of the lungs The complications associated with pheochromocytoma include hypertensive retinopathy and nephropathy, myocarditis, congestive heart failure (CHF), increased platelet aggregation, and stroke. Death can occur from shock, stroke, renal failure, dysrhythmias, or dissecting aortic aneurysm. Congestion heard on auscultation of the lungs is indicative of CHF. A urinary output of 50 mL/hr is an appropriate output; the nurse would become concerned if the output were less than 30 mL/hr. A coagulation time of 5 minutes is normal. A BUN level of 20 mg/dL is a normal finding.

A nurse is preparing to administer an injection of regular insulin. The vial of the regular insulin has been refrigerated. On inspection of the vial, the nurse finds that the medication is frozen. The nurse should: A. Wait for the insulin to thaw at room temperature B. Check the temperature settings of the refrigerator C. Discard the insulin and obtain another vial D. Rotate the vial between the hands until the medication becomes liquid

C. Discard the insulin and obtain another vial Insulin preparations are stable at room temperature for up to 1 month without significant loss of activity. Insulin should not be frozen. If the insulin is frozen, it should be discarded and the nurse should obtain another vial.

A college student is newly diagnosed with type 1 diabetes. She now has a headache, changes in her vision, and is anxious, but does not have her portable blood glucose monitor with her. Which action should the campus nurse advise her to take? A. Eat a piece of pizza. B. Drink some diet pop. C. Eat 15 g of simple carbohydrates. D. Take an extra dose of rapid-acting insulin.

C. Eat 15 g of simple carbohydrates. When the patient with type 1 diabetes is unsure about the meaning of the symptoms she is experiencing, she should treat herself for hypoglycemia to prevent seizures and coma from occurring. She should also be advised to check her blood glucose as soon as possible. The fat in the pizza and the diet pop would not allow the blood glucose to increase to eliminate the symptoms. The extra dose of rapid-acting insulin would further decrease her blood glucose.

A nurse is assisting in preparing a plan of care for the client with diabetes mellitus and plans to reinforce the client's understanding regarding the symptoms of hypoglycemia. Which symptoms will the nurse review? A. Slow pulse, lethargy, and warm, dry skin. B. Elevated pulse, lethargy, and warm, dry skin. C. Elevated pulse, shakiness and cool, clammy skin. D. Slow pulse, confusion, and increased urine output.

C. Elevated pulse, shakiness and cool, clammy skin. Symptoms of mild hypoglycemia include tachycardia; shakiness; and cool, clammy skin. Options 1, 2, and 4 are not symptoms of hypoglycemia.

The nurse is caring for a group of older patients in a long-term care setting. Which physical changes in the patients should the nurse investigate as signs of possible endocrine dysfunction? A. Absent reflexes, diarrhea, and hearing loss B. Hypoglycemia, delirium, and incontinence C. Fatigue, constipation, and mental impairment D. Hypotension, heat intolerance, and bradycardia

C. Fatigue, constipation, and mental impairment Changes of aging often mimic clinical manifestations of endocrine disorders. Clinical manifestations of endocrine dysfunction such as fatigue, constipation, or mental impairment in the older adult are often missed because they are attributed solely to aging.

When a nurse assesses a client receiving total parenteral nutrition (TPN), which laboratory value is most important for the nurse to monitor regularly? A. Albumin B. Calcium C. Glucose D. Alkaline phosphatase

C. Glucose TPN solutions contain high concentrations of glucose, so the blood glucose level is often monitored as often as q6h because of the risk for hyperglycemia (C). (A) is monitored periodically because an increase in the albumin level, a serum protein, is generally a desired effect of TPN. (B) may be added to TPN solutions, but calcium imbalances are not generally a risk during TPN administration. (D) may be decreased in the client with malnutrition who receives TPN, but abnormal values, reflecting liver or bone disorders, are not a common complication of TPN administration.

A nurse is preparing to provide instructions to a client with Addison's disease regarding diet therapy. The nurse understands that which of the following diets would likely be prescribed for this client? A. Low-protein diet. B. Low-sodium diet. C. High-sodium diet. D. Low-carb diet.

C. High-sodium diet. A high-sodium, high-complex carbohydrate, and high-protein diet will be prescribed for the client with Addison's disease. To prevent excess fluid and sodium loss, the client is instructed to maintain an adequate salt intake of up to 8 g of sodium daily and to increase salt intake during hot weather, before strenuous exercise, and in response to fever, vomiting, or diarrhea.

A nurse is providing instructions to a client newly diagnosed with diabetes mellitus. The nurse gives the client a list of the signs of hyperglycemia. Which of the following specific signs of this complication should be included on the list? A. Decreased urine output B. Profuse sweating C. Increased thirst D. Shakiness

C. Increased thirst The classic signs of hyperglycemia include polydipsia, polyuria, and polyphagia. Profuse sweating and shakiness would be noted in a hypoglycemic condition.

A patient, who is admitted with diabetes mellitus, has a glucose level of 380 mg/dL and a moderate level of ketones in the urine. As the nurse assesses for signs of ketoacidosis, which respiratory pattern would the nurse expect to find? A. Central apnea B. Hypoventilation C. Kussmaul respirations D. Cheyne-Stokes respirations

C. Kussmaul respirations In diabetic ketoacidosis, the lungs try to compensate for the acidosis by blowing off volatile acids and carbon dioxide. This leads to a pattern of Kussmaul respirations, which are deep and nonlabored. Central apnea occurs because the brain temporarily stops sending signals to the muscles that control breathing, which is unrelated to ketoacidosis. Hypoventilation and Cheyne-Stokes respirations do not occur with ketoacidosis.

The nurse is caring for a patient admitted with suspected hyperparathyroidism. Because of the potential effects of this disease on electrolyte balance, the nurse should assess this patient for what manifestation? A. Neurologic irritability B. Declining urine output C. Lethargy and weakness D. Hyperactive bowel sounds

C. Lethargy and weakness Hyperparathyroidism can cause hypercalcemia. Signs of hypercalcemia include muscle weakness, polyuria, constipation, nausea and vomiting, lethargy, and memory impairment. Neurologic irritability, declining urine output, and hyperactive bowel sounds do not occur with hypercalcemia.

Which nursing action would be appropriate to implement when a client has a diagnosis of pheochromocytoma? A. Weigh the client. B. Test the client's urine for glucose. C. Monitor the client's blood pressure. D. Palpate the client's skin to determine warmth.

C. Monitor the client's blood pressure. Hypertension is the major symptom that is associated with pheochromocytoma. The blood pressure status is monitored by taking the client's blood pressure. Glycosuria, weight loss, and diaphoresis are also clinical manifestations of pheochromocytoma, but hypertension is the major symptom.

During report, the nurse learns that a client with tumor lysis syndrome is receiving an IV infusion containing insulin. Which assessment should the nurse complete first? A. Review the client's history for diabetes mellitus. B. Observe the extremity distal to the IV site. C. Monitor the client's serum potassium and blood glucose levels. D. Evaluate the client's oxygen saturation and breath sounds.

C. Monitor the client's serum potassium and blood glucose levels. Clients with tumor lysis syndrome may experience hyperkalemia, requiring the addition of insulin to the IV solution to reduce the serum potassium level. It is most important for the nurse to monitor the client's serum potassium and blood glucose levels to ensure that they are not at dangerous levels (C). (A, B, and D) provide valuable assessment data but are of less priority than (C).

The surgeon was unable to spare a patient's parathyroid gland during a thyroidectomy. Which assessments should the nurse prioritize when providing postoperative care for this patient? A. Assessing the patient's white blood cell levels and assessing for infection B. Monitoring the patient's hemoglobin, hematocrit, and red blood cell levels C. Monitoring the patient's serum calcium levels and assessing for signs of hypocalcemia D. Monitoring the patient's level of consciousness and assessing for acute delirium or agitation

C. Monitoring the patient's serum calcium levels and assessing for signs of hypocalcemia Loss of the parathyroid gland is associated with hypocalcemia. Infection and anemia are not associated with loss of the parathyroid gland, whereas cognitive changes are less pronounced than the signs and symptoms of hypocalcemia.

The nurse is caring for a 36-year-old woman with possible hypoparathyroidism after a thyroidectomy. It is most appropriate for the nurse to assess for which clinical manifestations? A. Polyuria, polydipsia, and weight loss B. Cardiac dysrhythmias and hypertension C. Muscle spasms and hyperactive deep tendon reflexes D. Hyperpigmentation, skin ulcers, and peripheral edema

C. Muscle spasms and hyperactive deep tendon reflexes Common assessment abnormalities associated with hypoparathyroidism include tetany (muscle spasms) and increased deep tendon reflexes. Hyperpigmentation is associated with Addison's disease. Skin ulcers occur in patient with diabetes. Edema is associated with hypothyroidism. Polyuria and polydipsia occur in patients with diabetes mellitus or diabetes insipidus. Weight loss occurs in hyperthyroidism or diabetic ketoacidosis. Hypertension and cardiac dysrhythmias may be caused by hyperthyroidism, hyperparathyroidism, or pheochromocytoma.

The laboratory report of a client reveals increased serum cholesterol levels. Which other finding indicates growth hormone deficiency in the client? A. Scalp alopecia B. Intolerance to cold C. Pathological fractures D. Increased urine output

C. Pathological fractures Growth hormone deficiency results in thinning of bones and increases the risk for pathological fractures. Thyrotropin deficiency results in scalp alopecia and intolerance to cold. Marked increase in the volume of urine output is a sign of diabetes insipidus caused by vasopressin deficiency.

What is a nursing priority in the care of a patient with a diagnosis of hypothyroidism? A. Providing a dark, low-stimulation environment B. Closely monitoring the patient's intake and output C. Patient teaching related to levothyroxine (Synthroid) D. Patient teaching related to radioactive iodine therapy

C. Patient teaching related to levothyroxine (Synthroid) A euthyroid state is most often achieved in patients with hypothyroidism by the administration of levothyroxine (Synthroid). It is not necessary to carefully monitor intake and output, and low stimulation and radioactive iodine therapy are indicated in the treatment of hyperthyroidism.

A patient has been taking oral prednisone for the past several weeks after having a severe reaction to poison ivy. The nurse has explained the procedure for gradual reduction rather than sudden cessation of the drug. What is the rationale for this approach to drug administration? A. Prevention of hypothyroidism B. Prevention of diabetes insipidus C. Prevention of adrenal insufficiency D. Prevention of cardiovascular complications

C. Prevention of adrenal insufficiency Sudden cessation of corticosteroid therapy can precipitate life-threatening adrenal insufficiency. Diabetes insipidus, hypothyroidism, and cardiovascular complications are not common consequences of suddenly stopping corticosteroid therapy.

The newly diagnosed patient with type 2 diabetes has been prescribed metformin (Glucophage). What should the nurse tell the patient to best explain how this medication works? A. Increases insulin production from the pancreas. B. Slows the absorption of carbohydrate in the small intestine. C. Reduces glucose production by the liver and enhances insulin sensitivity. D. Increases insulin release from the pancreas, inhibits glucagon secretion, and decreases gastric emptying.

C. Reduces glucose production by the liver and enhances insulin sensitivity. Metformin is a biguanide that reduces glucose production by the liver and enhances the tissue's insulin sensitivity. Sulfonylureas and meglitinides increase insulin production from the pancreas. α-glucosidase inhibitors slow the absorption of carbohydrate in the intestine. Glucagon-like peptide receptor agonists increase insulin synthesis and release from the pancreas, inhibit glucagon secretion, and decrease gastric emptying.

A client newly diagnosed with type 1 diabetes is taught to exercise on a regular basis. What is the primary reason for instruction on exercise? A. To decrease insulin sensitivity B. To stimulate glucagon production C. To improve the cellular uptake of glucose D. To reduce metabolic requirements for glucose

C. To improve the cellular uptake of glucose Exercise increases the metabolic rate, and glucose is needed for cellular metabolism; therefore, excess glucose is consumed during exercise. Regular vigorous exercise increases cell sensitivity to insulin. Glucagon action raises blood glucose but does not affect cell uptake or use of glucose. Cellular requirements for glucose increase with exercise.

A nurse caring for a client scheduled for a transsphenoidal hypophysectomy to remove a tumor in the pituitary gland assists to develop a plan of care for the client. The nurse suggests including which specific information in the preoperative teaching plan? A. Hair will need to be shaved. B. Deep breathing and coughing will be needed after surgery. C. Toothbrushing will not be permitted for at least 2 weeks following surgery. D. Spinal anesthesia is used.

C. Toothbrushing will not be permitted for at least 2 weeks following surgery. Based on the location of the surgical procedure, spinal anesthesia would not be used. In addition, the hair would not be shaved. Although coughing and deep breathing are important, specific to this procedure is avoiding toothbrushing to prevent disruption of the surgical site. Also, coughing may disrupt the surgical site.

When instructing a patient regarding a urine study for free cortisol, what is most important for the nurse to tell the patient? A. Save the first voided urine in the morning. B. Maintain a high-sodium diet 3 days before collection. C. Try to avoid stressful situations during the collection period. D. Complete at least 30 minutes of exercise before collecting the urine sample.

C. Try to avoid stressful situations during the collection period. A urine study for free cortisol requires a 24-hour urine collection. The patient should be instructed to avoid stressful situations and excessive physical exercise that could unduly increase cortisol levels. The patient should also maintain a low-sodium diet before and during the urine collection period.

When the nurse is teaching a client who has been newly diagnosed with type 1 diabetes mellitus, which statement by the client would indicate that teaching has been effective? A. "I will stop taking my insulin if I'm too sick to eat." B. "I will decrease my insulin dose during times of illness." C. "I will adjust my insulin dose according to the level of glucose in my urine." D. "I will notify my health care provider if my blood glucose level is greater than 250 mg/dL."

D. "I will notify my health care provider if my blood glucose level is greater than 250 mg/dL." During illness, the client should monitor the blood glucose level, and he or she should notify the health care provider (HCP) if the level is greater than 250 mg/dL. Insulin should never be stopped. In fact, insulin may need to be increased during times of illness. Doses should not be adjusted without the HCP's advice.

The nurse is teaching a patient with type 2 diabetes mellitus about exercise to help control his blood glucose. The nurse knows the patient understands when the patient elicits which exercise plan? A. "I want to go fishing for 30 minutes each day; I will drink fluids and wear sunscreen." B. "I will go running each day when my blood sugar is too high to bring it back to normal." C. "I will plan to keep my job as a teacher because I get a lot of exercise every school day." D. "I will take a brisk 30-minute walk 5 days per week and do resistance training 3 times a week."

D. "I will take a brisk 30-minute walk 5 days per week and do resistance training 3 times a week." The best exercise plan for the person with type 2 diabetes is for 30 minutes of moderate activity 5 days per week and resistance training 3 times a week. Brisk walking is moderate activity. Fishing and teaching are light activity, and running is considered vigorous activity.

A patient with a severe pounding headache has been diagnosed with hypertension. However, the hypertension is not responding to traditional treatment. What should the nurse expect as the next step in management of this patient? A. Administration of β-blocker medications B. Abdominal palpation to search for a tumor C. Administration of potassium-sparing diuretics D. A 24-hour urine collection for fractionated metanephrines

D. A 24-hour urine collection for fractionated metanephrines Pheochromocytoma should be suspected when hypertension does not respond to traditional treatment. The 24-hour urine collection for fractionated metanephrines is simple and reliable with elevated values in 95% of people with pheochromocytoma. In a patient with pheochromocytoma preoperatively an α-adrenergic receptor blocker is used to reduce BP. Abdominal palpation is avoided to avoid a sudden release of catecholamines and severe hypertension. Potassium-sparing diuretics are not needed. Most likely they would be used for hyperaldosteronism, which is another cause of hypertension.

The hypothalamus secretes releasing hormones and inhibiting hormones. What is the target tissue of these releasing hormones and inhibiting hormones? A. Pineal B. Adrenal cortex C. Posterior pituitary D. Anterior pituitary

D. Anterior pituitary The anterior pituitary is the target tissue of the releasing hormones (corticotropin releasing hormone, thyrotropin releasing hormone, growth hormone releasing factor, gonadotropin releasing hormone, prolactin releasing factor) and the inhibiting hormones (somatostatin, prolactin inhibiting factor). These hormones release or inhibit other hormones that affect the thyroid, adrenal cortex, pancreas, reproductive organs, and all body cells. The pineal gland is not directly affected by the releasing and inhibiting hormones from the hypothalamus. The posterior pituitary releases antidiuretic hormone (ADH) in response to plasma osmolality changes that is not directly affected by the hypothalamus hormones.

Which hormonal deficiency causes diabetes insipidus in a client? A. Prolactin B. Thyrotropin C. Luteinizing hormone (LH) D. Antidiuretic hormone (ADH)

D. Antidiuretic hormone (ADH) ADH deficiency causes diabetes insipidus. Decreased levels of prolactin may cause decreased amounts of milk secretion after birth. Decreased levels of thyrotropin cause hypothyroidism, weight gain, and lethargy. LH deficiency causes menstrual abnormalities, decreased libido, and breast atrophy.

The nurse caring for a client who has had a subtotal thyroidectomy reviews the plan of care and determines which problem is the priority for this client in the immediate postoperative period? A. Dehydration B. Infection C. Urinary retention D. Bleeding

D. Bleeding Hemorrhage is one of the most severe complications that can occur following thyroidectomy. The nurse must frequently check the neck dressing for bleeding and monitor vital signs to detect early signs of hemorrhage, which could lead to shock. T3 and T4 do not regulate fluid volumes in the body. Infection is a concern for any postoperative client but is not the priority in the immediate postoperative period. Urinary retention can occur in postoperative clients as a result of medication and anesthesia but is not the priority from the options provided.

A nurse enters the room of a client with type 1 diabetes mellitus and finds the client difficult to arouse. The client's skin is warm and flushed, and the pulse and respiratory rate are elevated from the client's baseline. The nurse would immediately: A. Prepare for the administration of an insulin drip. B. Give the client a glass of orange juice. C. Prepare for the administration of a bolus of 50% dextrose. D. Check the client's capillary blood glucose.

D. Check the client's capillary blood glucose. The nurse must first obtain a blood glucose reading to determine the client's problem. Options 2 and 3 would be implemented as needed in the treatment of hypoglycemia. Insulin therapy is guided by blood glucose measurement.

A nurse is caring for a client who has just returned from the postanesthesia care unit after having a thyroidectomy. Which action has priority during the first 24 hours after surgery when the nurse is concerned about thyroid storm? A. Performing range-of-motion exercises B. Humidifying the room air continuously C. Assessing for hoarseness every two hours D. Checking vital signs every two hours after they stabilize

D. Checking vital signs every two hours after they stabilize Checking vital signs helps detect complications such as thyrotoxic crisis, hemorrhage, and respiratory obstruction that may occur early in the postoperative period. Range-of-motion exercises should not begin until two to four days postoperatively because they can disrupt the suture line. A humidifier can contribute to the spread of bacteria and infection and is contraindicated. Hoarseness and voice weakness usually are temporary and not life threatening; the priority is to observe for thyroid storm, hemorrhage, and respiratory obstruction.

The nurse caring for a patient hospitalized with diabetes mellitus would look for which laboratory test result to obtain information on the patient's past glucose control? A. Prealbumin level B. Urine ketone level C. Fasting glucose level D. Glycosylated hemoglobin level

D. Glycosylated hemoglobin level A glycosylated hemoglobin level detects the amount of glucose that is bound to red blood cells (RBCs). When circulating glucose levels are high, glucose attaches to the RBCs and remains there for the life of the blood cell, which is approximately 120 days. Thus the test can give an indication of glycemic control over approximately 2 to 3 months. The prealbumin level is used to establish nutritional status and is unrelated to past glucose control. The urine ketone level will only show that hyperglycemia or starvation is probably currently occurring. The fasting glucose level only indicates current glucose control.

The client's pituitary gland must be removed. Which surgery will the client undergo? A. Mastectomy B. Prostatectomy C. Thyroidectomy D. Hypophysectomy

D. Hypophysectomy A hypophysectomy is the surgical removal of the pituitary gland or its tumor. A mastectomy is the surgical removal of breast tissue. A prostatectomy is the surgical removal of the prostate gland. A thyroidectomy is the surgical removal of the thyroid gland.

A client is brought to the emergency department in an unresponsive state, and a diagnosis of hyperglycemic hyperosmolar nonketotic syndrome (HHNS) is made. The nurse who is assisting to care for the client obtains which of the following immediately in preparation for the treatment of this syndrome? A. NPH insulin B. A nasal cannula C. Intravenous (IV) infusion of sodium bicarbonate D. IV infusion of normal saline

D. IV infusion of normal saline The primary goal of treatment is to rehydrate the client to restore fluid volume and to correct electrolyte deficiency. IV fluid replacement is similar to that administered in diabetic ketoacidosis (DKA) and begins with IV infusion of normal saline. Regular insulin, not NPH insulin would be administered. The use of sodium bicarbonate to correct acidosis is avoided because it can precipitate a further drop in serum potassium levels. A nasal cannula for oxygen administration is not necessarily required to treat HHNS.

A client with Graves' disease has exophthalmos and is experiencing photophobia. Which intervention would best assist the client with this problem? A. Administering methimazole every 8 hours B. Lubricating the eyes with tap water every 2 to 4 hours C. Instructing the client to avoid straining or heavy lifting D. Obtaining dark glasses for the client

D. Obtaining dark glasses for the client Because photophobia (light intolerance) accompanies this disorder, dark glasses are helpful in alleviating the symptom. Medical therapy for Graves' disease does not help alleviate the clinical manifestation of exophthalmos. Other interventions may be used to relieve the drying that occurs from not being able to completely close the eyes; however, the question is asking what the nurse can do for photophobia. Tap water, which is hypotonic, could actually cause more swelling to the eye because it could pull fluid into the interstitial space. In addition, the client is at risk for developing an eye infection because the solution is not sterile. There is no need to prevent straining with exophthalmos.

A nurse is caring for a client experiencing thyroid storm. Which of the following would be a priority concern for this client? A. Inability to cope with the treatment plan B. Lack of sexual drive C. Self-consciousness about body appearance D. Potential for cardiac disturbances

D. Potential for cardiac disturbances Clients in thyroid storm are experiencing a life-threatening event, which is associated with uncontrolled hyperthyroidism. It is characterized by high fever, severe tachycardia, delirium, dehydration, and extreme irritability. The signs and symptoms of the disorder develop quickly, and therefore emergency measures must be taken to prevent death. These measures include maintaining hemodynamic status and patency of airway as well as providing adequate ventilation. Options 1, 2, and 3 are not a priority in the care of the client in thyroid storm.

A nurse is collecting data regarding a client after a thyroidectomy and notes that the client has developed hoarseness and a weak voice. Which nursing action is appropriate? A. Check for signs of bleeding. B. Administer calcium gluconate. C. Notify the health care provider immediately. D. Reassure the client that this is usually a temporary condition.

D. Reassure the client that this is usually a temporary condition. Weakness and hoarseness of the voice can occur as a result of trauma of the laryngeal nerve. If this develops, the client should be reassured that the problem will subside in a few days. Unnecessary talking should be discouraged. It is not necessary to notify the registered nurse immediately. These signs do not indicate bleeding or the need to administer calcium gluconate.

A client with type 2 diabetes takes metformin (Glucophage) daily. The client is scheduled for major surgery requiring general anesthesia the next day. The nurse anticipates which approach to manage the client's diabetes best while the client is NPO during the perioperative period? A. NPO except for metformin and regular snacks B. NPO except for oral antidiabetic agent C. Novolin N insulin subcutaneously twice daily D. Regular insulin subcutaneously per sliding scale

D. Regular insulin subcutaneously per sliding scale Regular insulin dosing based on the client's blood glucose levels (sliding scale) is the best method to achieve control of the client's blood glucose while the client is NPO and coping with the major stress of surgery (D). (A) increases the risk of vomiting and aspiration. (B and C) provide less precise control of the blood glucose level.

The nurse receives the client's next scheduled bag of TPN labeled with the additive NPH insulin. Which action should the nurse implement? A. Hang the solution at the current rate. B. Refrigerate the solution until needed. C. Prepare the solution with new tubing. D. Return the solution to the pharmacy.

D. Return the solution to the pharmacy. Only regular insulin is administered by the IV route, so the TPN solution containing NPH insulin should be returned to the pharmacy (D). (A, B, and C) are not indicated because the solution should not be administered.

Which neurologic manifestation in a client is associated with hyperthyroidism? A. Confusion B. Hearing loss C. Exophthalmos D. Slowness of speech

C. Exophthalmos In hyperthyroidism, edema in the extraocular muscles and increased fatty tissue behind the eye leads to exophthalmos. Confusion, hearing loss, and slowness of speech are caused by hypothyroidism.

An Asian American client is experiencing a fever. The nurse recognizes that the client is likely to self-treat the disorder, using which method? A. Prayer B. Magnetic therapy C. Foods considered to be yin D. Foods considered to be yang

C. Foods considered to be yin

The nurse is caring for an unconscious client who is experiencing persistent hyperthermia with no signs of infection. The nurse interprets that the hyperthermia may be related to damage to the client's thermoregulatory center in which structure? A. Cerebrum B. Cerebellum C. Hippocampus D. Hypothalamus

4. Hypothalamus

What are the most common hormones produced in excess with hyperpituitarism? Select all that apply. A. Prolactin B. Growth hormone C. Luteinizing hormone D. Antidiuretic hormone E. Melanocyte-stimulating hormone

A, B. The most common hormones produced in excess with hyperpituitarism are prolactin and growth hormone. Excessive stimulation of luteinizing hormone and antidiuretic hormone is also associated with hyperpituitarism, but less commonly than prolactin and growth hormone. Secretion of melanocyte-stimulating hormone stimulates adrenocorticotropic hormone, which indirectly stimulates the pituitary gland, thus leading to hyperpituitarism.

Which exogenous conditions are responsible for increased cortisol secretion? Select all that apply. A. Asthma B. Adrenal adenomas C. Cancer chemotherapy D. Organ transplantation E. Carcinomas of the lung

A, C, D. Asthma, cancer chemotherapy, and organ transplantation are the exogenous conditions responsible for increased cortisol secretion. This is because when adrenocorticotropic hormone or glucocorticoids are administered for the treatment of these conditions, it promotes cortisol secretion in the client, thus causing Cushing's syndrome. Adrenal adenomas and carcinomas of the lung are endogenous conditions in which the endogenous secretions increase the cortisol levels in the client, thereby also causing Cushing's syndrome.

The nurse is teaching a client newly diagnosed with diabetes about the importance of glucose monitoring. Which blood glucose levels should the nurse identify as hypoglycemia? A. 68 mg/dL B. 78 mg/dL C. 88 mg/dL D. 96 mg/dL

A. 68 mg/dL Normal blood glucose level for an adult is 72-108 mg/dL (4-6 mmol/L). Clients who have blood glucose levels below 72 mg/dL (4 mmol/L) may experience hypoglycemia; 78 mg/dL (4.3 mmol/L), 88 mg/dL (4.9 mmol/L), and 98 mg/dL (5.4 mmol/L) are normal blood glucose levels.

The nurse is caring for the client post-transsphenoidal hypophysectomy. When assessing the client, the nurse observes clear drainage from the nares. What could be the cause of this drainage? A. A cerebral spinal fluid leak from an opening to the brain. B. A normal occurrence for this client's procedure. C. The client is developing an infection. D. The client may have had a cold preoperatively, and the nurse will continue to monitor.

A. A cerebral spinal fluid leak from an opening to the brain. Transsphenoidal hypophysectomy is removal of the pituitary gland. This procedure is close to the brain. Clear drainage from the nares could indicate a cerebral spinal fluid (CSF) leak. The nurse should contact the primary healthcare provider and send the drainage to the laboratory for glucose evaluation. If the glucose level is greater than 30 mg/dL, this would indicate a CSF leak. This is not a normal occurrence postoperatively for this procedure. Clear drainage would not indicate an infection.

A nurse is planning to teach facts about hyperglycemia to a client with diabetes. What information should the nurse include in the discussion about what causes diabetic acidosis? A. Breakdown of fat store for energy B. Ingestion of too many highly acidic foods C. Excessive secretion of endogenous insulin D. Increased amounts of cholesterol in the extracellular compartment

A. Breakdown of fat store for energy' In the absence of insulin, which facilitates the transport of glucose into cells, the body breaks down proteins and fats to supply energy; ketones, a by-product of fat metabolism, accumulate, causing metabolic acidosis (pH below 7.35). The pH of food ingested has no effect on the development of acidosis. The opposite of excessive secretion of endogenous insulin is true. Cholesterol level has no effect on the development of acidosis.

What intervention should the nurse implement when caring for a client 24 hours post-thyroidectomy? A. Check the back and sides of the operative site. B. Support the head during mild range-of-motion exercises. C. Encourage the client to ventilate feelings about the surgery. D. Advise the client that regular activities can be resumed immediately.

A. Check the back and sides of the operative site. Bleeding may occur, and blood will pool in the back of the neck because the blood will flow via gravity. ROM exercises will increase pain and put tension on the suture line. Talking should be avoided in the immediate postoperative period, except to assess for a change in pitch or tone, which may indicate laryngeal nerve damage. Activity should be resumed gradually and frequent rest periods encouraged.

Which condition is characterized by hemorrhage after a pregnant female delivers? A. Sheehan's syndrome B. Addison's syndrome C. Cushing's syndrome D. Schwartz-Bartter syndrome

A. Sheehan's syndrome A pituitary infarction is caused by postpartum hemorrhaging; this condition is known as Sheehan's syndrome. Cushing's syndrome is manifested by moon face, truncal obesity, and hypertension. Addison's disease is manifested by hyperkalemia, hypotension, and hypoglycemia. Schwartz-Bartter syndrome, also called syndrome of inappropriate antidiuretic hormone, is manifested by loss of appetite, nausea, and vomiting.

A nurse is caring for a client with a diagnosis of type 1 diabetes who has developed diabetic coma. Which element excessively accumulates in the blood to precipitate the signs and symptoms associated with this condition? A. Sodium bicarbonate, causing alkalosis B. Ketones as a result of rapid fat breakdown, causing acidosis C. Nitrogen from protein catabolism, causing ammonia intoxication D. Glucose from rapid carbohydrate metabolism, causing drowsiness

B. Ketones as a result of rapid fat breakdown, causing acidosis Ketones are produced when fat is broken down for energy. Although rarely used, sodium bicarbonate may be administered to correct the acid-base imbalance resulting from ketoacidosis; acidosis is caused by excess acid, not excess base bicarbonate. Diabetes does not interfere with removal of nitrogenous wastes. Carbohydrate metabolism is impaired in the client with diabetes.

The serum potassium level of a client who has diabetic ketoacidosis is 5.4 mEq/L (5.4 mmol/L). What would the nurse expect to see on the ECG tracing monitor? A. Abnormal P waves and depressed T waves B. Peaked T waves and widened QRS complexes C. Abnormal Q waves and prolonged ST segments D. Peaked P waves and an increased number of T waves

B. Peaked T waves and widened QRS complexes Potassium is the principal intracellular cation, and during ketoacidosis it moves out of cells into the extracellular compartment to replace potassium lost as a result of glucose-induced osmotic diuresis; overstimulation of the cardiac muscle results. The T wave is depressed in hypokalemia. Initially, the QT segment is short, and as the potassium level rises, the QRS complex widens. P waves are abnormal because the PR interval may be prolonged and the P wave may be lost; however, the T wave is peaked, not depressed. The ST segment becomes depressed. The PR interval is prolonged, and the P wave may be lost. QRS complexes and thus T waves become irregular, and the rate does not necessarily change.

The nurse has applied a hypothermia blanket to a client with a fever. A priority for the nurse is to inspect the skin frequently to detect which complication of hypothermia blanket use? A. Frostbite B. Skin breakdown C. Venous insufficiency D. Arterial insufficiency

B. Skin breakdown

After reviewing the client's laboratory reports, the physician concludes that the client has primary hypofunction of the adrenal gland. Which clinical manifestation is likely to be observed in that client? A. Edema at extremities B. Uneven patches of pigment loss C. Reddish-purple stretch marks on the abdomen D. "Buffalo hump" between shoulders on the back

B. Uneven patches of pigment loss Vitiligo is manifested by the presence of large patchy areas of pigment loss. This is mainly caused by primary hypofunction of the adrenal gland. Presence of edema at extremities indicates fluid and electrolyte imbalances mainly observed in a client with thyroid problems. Presence of reddish-purple stretch marks on the abdomen and "buffalo hump" between shoulders on the back of the neck often indicates excessive adrenocortical secretions.

Which term should the nurse use in a report to describe the absence of menstrual periods in a 35-year-old non-pregnant client? A. Rhinorrhea B. Menopause C. Amenorrhea D. Dyspareunia

C. Amenorrhea The absence of menstrual periods in a non-pregnant client less than 55 years old is called amenorrhea. Rhinorrhea is an allergic state that is manifested by a runny nose. Menopause is cessation of menstruation after 55 years of age. Dyspareunia is pain during sexual intercourse.

While obtaining the client's health history, which factor does the nurse identify that predisposes the client to type 2 diabetes? A. Having diabetes insipidus B. Eating low-cholesterol foods C. Being 20 pounds overweight D. Drinking a daily alcoholic beverage

C. Being 20 pounds overweight Excessive body weight is a known predisposing factor to type 2 diabetes; the exact relationship is unknown. Diabetes insipidus is caused by too little antidiuretic hormone (ADH) and has no relationship to type 2 diabetes. High-cholesterol diets and atherosclerotic heart disease are associated with type 2 diabetes. Alcohol intake is not known to predispose a person to type 2 diabetes.

The nurse in the newborn nursery is preparing to complete an initial assessment on a newborn infant who was just admitted to the nursery. The nurse should place a warm blanket on the examining table to prevent heat loss in the infant caused by which method? A. Radiation B. Convection C. Conduction D. Evaporation

C. Conduction

A client arrives at the postpartum unit after delivery of her infant. On performing an assessment, the nurse notes that the client is shaking uncontrollably. Which nursing action would be appropriate? A. Massage the fundus. B. Contact the health care provider. C. Cover the client with a warm blanket. D. Place the client in Trendelenburg's position.

C. Cover the client with a warm blanket.

Which drug would be effective for the treatment of pituitary Cushing's syndrome? A. Mitotane B. Cabergoline C. Cyproheptadine D. Bromocriptine mesylate

C. Cyproheptadine Cyproheptadine is effective for the treatment of pituitary Cushing's syndrome. Mitotane is prescribed for the treatment of adrenal Cushing's syndrome. Cabergoline and bromocriptine mesylate are effective for the treatment of hyperpituitarism.

The nurse assisted with the delivery of a newborn. Which nursing action is most effective in preventing heat loss by evaporation? A. Warming the crib pad B. Closing the doors to the room C. Drying the infant with a warm blanket D. Turning on the overhead radiant warmer

C. Drying the infant with a warm blanket

Four hours after surgery, the blood glucose level of a client who has type 1 diabetes is elevated. What intervention should the nurse implement? A. Administer an oral hypoglycemic B. Institute urine glucose monitoring C. Give supplemental doses of regular insulin D. Decrease the rate of the intravenous infusion

C. Give supplemental doses of regular insulin The blood glucose level needs to be reduced; regular insulin begins to act in 30 to 60 minutes. The client has type 1, not type 2, diabetes, and an oral hypoglycemic will not be effective. Blood glucose levels are far more accurate than urine glucose levels. The rate may be increased because polyuria often accompanies hyperglycemia.

A nurse is reviewing several charts. Which condition is an autoimmune disorder? A. Addison's disease B. Cushing's syndrome C. Hashimoto's disease D. Sheehan's syndrome

C. Hashimoto's disease Hashimoto's disease is an autoimmune disorder, wherein the immune system attacks the thyroid gland. Addison's disease is caused by adrenal insufficiency. Cushing's syndrome is caused by increased body levels of cortisol. Sheehan's syndrome is hemorrhage-associated hypopituitarism after delivery of a child.

What will the nurse expect diagnostic studies of a client with Cushing syndrome to indicate? A. Moderately increased serum potassium levels B. Increased numbers of eosinophils in the blood C. High levels of 17-ketosteroids in a 24-hour urine test D. Normal to low levels of adrenocorticotropic hormone (ACTH)

C. High levels of 17-ketosteroids in a 24-hour urine test High levels of 17-ketosteroids in a 24-hour urine test is a urinary metabolite of steroid hormones that are excreted in large amounts in hyperaldosteronism. With aldosterone hypersecretion, sodium is retained and potassium is excreted, resulting in hypernatremia and hypokalemia. With Cushing syndrome, the eosinophil count is decreased, not increased. ACTH levels usually are high in Cushing syndrome.

A nurse is caring for a client with endocrine problems. Which lab finding will alert the nurse that aldosterone will be released? A. Hypokalemia B. Hypoglycemia C. Hyponatremia D. Hypochloremia

C. Hyponatremia Hyponatremia stimulates the secretion of aldosterone. Hypoglycemia inhibits the secretion of insulin. Hyperkalemia, not hypokalemia, stimulates the secretion of aldosterone. Hypochloremia is associated with increased levels of antidiuretic hormone.

After surgical clipping of a ruptured cerebral aneurysm, a client develops the syndrome of inappropriate secretion of antidiuretic hormone (SIADH). What manifestations are exhibited with excessive levels of antidiuretic hormone? A. Increased blood urea nitrogen (BUN) and hypotension B. Hyperkalemia and poor skin turgor C. Hyponatremia and decreased urine output D. Polyuria and increased specific gravity of urine

C. Hyponatremia and decreased urine output Antidiuretic hormone (ADH) causes water retention, resulting in a decreased urine output and dilution of serum electrolytes. Blood volume may increase, causing hypertension. Diluting the nitrogenous wastes in the blood decreases rather than increases the BUN. Water retention dilutes electrolytes. The client is overhydrated rather than underhydrated, so turgor is not poor. ADH acts on the nephron to cause water to be reabsorbed from the glomerular filtrate, leading to reduced urine volume. The specific gravity of urine is elevated as a result of increased concentration.

A nurse is caring for a client newly diagnosed with type 1 diabetes. When the primary healthcare provider tries to regulate this client's insulin regimen, the client experiences episodes of hypoglycemia and hyperglycemia, and 15 g of a simple sugar is prescribed. What is the reason this is administered when a client experiences hypoglycemia? A. Inhibits glycogenesis B. Stimulates release of insulin C. Increases blood glucose levels D. Provides more storage of glucose

C. Increases blood glucose levels A simple sugar provides glucose to the blood for rapid action. It does not inhibit glycogenesis. It does not stimulate the release of insulin. It does not stimulate the storage of glucose.

A nurse is caring for a client with type 1 diabetes who is experiencing a fluid imbalance. Which fluid shift associated with diabetes should the nurse take into consideration when assessing this client? A. Intravascular to interstitial as a result of glycosuria B. Extracellular to interstitial as a result of hypoproteinemia C. Intracellular to intravascular as a result of hyperosmolarity D. Intercellular to intravascular as a result of increased hydrostatic pressure

C. Intracellular to intravascular as a result of hyperosmolarity The osmotic effect of hyperglycemia pulls fluid from the cells, resulting in cellular dehydration. Hyperglycemia pulls fluid from the interstitial compartment to the intravascular compartment. Interstitial fluid is part of the extracellular compartment; the osmotic pull of glucose exceeds that of other osmotic forces. An increase in hydrostatic pressure results in an intravascular-to-interstitial shift.

Which clinical manifestation is found in a client with a deficiency of adrenocorticotropic hormone? A. Anovulation B. Dehydration C. Malaise and lethargy D. Menstrual abnormalities

C. Malaise and lethargy Malaise is a general feeling of discomfort or illness and lethargy is a lack of energy. A client with deficiency of adrenocorticotropic hormone may experience malaise and lethargy. Adrenocorticotropic hormone deficiency is not associated with anovulation, dehydration, and menstrual abnormalities. Anovulation (ovaries do not release an oocyte during the menstrual cycle) occurs due to deficiency of gonadotropins. Dehydration is a result of deficiency of antidiuretic hormone. The deficiency of thyroid-stimulating hormone may result in menstrual abnormalities.

Which condition results in elevated serum adrenocorticotropic hormone (ACTH) and urine cortisol levels? A. Diabetes Insipidus B. Adrenal Cushing's syndrome C. Pituitary Cushing's syndrome D. Syndrome of inappropriate antidiuretic hormone

C. Pituitary Cushing's syndrome In pituitary Cushing's syndrome, urine cortisol and serum adrenocorticotropic hormone levels are raised. Diabetes insipidus is the result of decreased levels of antidiuretic hormone and is not associated with cortisol and ACTH levels. Adrenal Cushing's syndrome is caused by chronic steroid use, so the client will have increased urine cortisol and decreased ACTH levels. Syndrome of inappropriate antidiuretic hormone is the result of elevated levels of antidiuretic hormone and is not related with the ACTH and cortisol levels.

Which does the nurse state is a secondary cause of adrenal insufficiency? A. Hemorrhage B. Tuberculosis C. Pituitary tumors D. Metastatic cancer

C. Pituitary tumors Adrenal insufficiency is also called Addison's disease. Secondary causes of adrenal insufficiency include pituitary tumors. Primary causes, which are responsible for adrenal insufficiency, include hemorrhage, tuberculosis, and metastatic cancer.

A client with a neurological problem is experiencing hyperthermia. Which measure would be least appropriate for the nurse to use in trying to lower the client's body temperature? A. Giving tepid sponge baths. B. Applying a hypothermia blanket. C. Placing ice packs in the axilla and groin areas. D. Administering acetaminophen (Tylenol) per protocol.

C. Placing ice packs in the axilla and groin areas.

A client who has had a subtotal thyroidectomy does not understand how hypothyroidism can develop when the problem was initially hyperthyroidism. On what fact should the nurse base her response? A. Hypothyroidism is a gradual slowing of the body's function. B. There will be a decrease in pituitary thyroid-stimulating hormone (TSH). C. There may not be enough thyroid tissue to supply adequate thyroid hormone. D. Atrophy of tissue remaining after surgery reduces secretion of thyroid hormones.

C. There may not be enough thyroid tissue to supply adequate thyroid hormone. After a subtotal thyroidectomy the thyroxine output may be inadequate to maintain an appropriate metabolic rate. Hypothyroidism is a decrease in thyroid functioning, not a slowing of the entire body's functions. In hypothyroidism the level of TSH from the pituitary usually is increased. Atrophy of the remaining thyroid tissue does not occur.

A client with type 2 diabetes, who is taking an oral hypoglycemic agent, is to have a serum glucose test early in the morning. The client asks the nurse, "What do I have to do to prepare for this test?" Which statement by the nurse reflects accurate information? A. "Eat your usual breakfast." B. "Have clear liquids for breakfast." C. "Take your medication before the test." D. "Do not ingest anything before the test."

D. "Do not ingest anything before the test." Fasting before the test is indicated for accurate and reliable results; food before the test will increase serum glucose levels through metabolism of the nutrients. Food should not be ingested before the test; food will increase the serum glucose level, negating accuracy of the test. Instructing the client to have clear liquids for breakfast is inappropriate; some clear fluids contain simple carbohydrates, which will increase the serum glucose level. Medications are withheld before the test because of their influence on the serum glucose level.

A nurse is monitoring a client's laboratory results for a fasting plasma glucose level. Within which range of a fasting plasma glucose level does the nurse conclude that a client is considered to be diabetic? A. 40 to 60 mg/dL B. 80 to 99 mg/dL C. 100 to 125 mg/dL D. 126 to 140 mg/dL

D. 126 to 140 mg/dL Results in the range 126 to 140 mg/dL (7.0 to 7.8 mmol/L) indicate diabetes. Results in the range 40 to 60 mg/dL (2.2 to 3.3 mmol/L) indicate hypoglycemia. Results in the range 80 to 99 mg/dL (4.5 to 5.5 mmol/L) are considered expected (normal). Results in the range 100 to 125 mg/dL (5.6 to 6.9 mmol/L) indicate prediabetes according to the American Diabetes Association.

A client has undergone nasal hypophysectomy surgery. During post-operative care, which finding indicates cerebrospinal leakage? A. Dry mouth B. Rigidity of neck muscles C. Fall in blood pressure upon standing D. A yellow edge around nasal discharge

D. A yellow edge around nasal discharge Nasal hypophysectomy is a surgical procedure performed to treat hyperpituitarism due to pituitary gland tumors. During postoperative care and follow-up, the appearance of light-yellow at the edge of otherwise clear nasal discharge in the dressing indicates leakage of cerebrospinal fluid (CSF). This is called the "halo sign" and is indicative of a CSF leak. Dry mouth after nasal hypophysectomy is normal because the client breathes through the mouth due to the nasal packing. Neck rigidity could be an indication of infection, such as meningitis following the surgery. A fall in blood pressure upon standing is called orthostatic hypotension and is a side effect of bromocriptine.

The nurse is caring for a client with acquired immunodeficiency syndrome (AIDS) who is experiencing night fever and night sweats. Which nursing intervention would be the least helpful in managing this symptom? A. Keep liquids at the bedside. B. Make sure the pillow has a plastic cover. C. Keep a change of bed linens nearby in case they are needed. D. Administer an antipyretic after the client has a spike in temperature.

D. Administer an antipyretic after the client has a spike in temperature.

A client is admitted for hypertension, and serum electrolyte studies have yielded abnormal results. The scheduled workup includes a scan for an aldosteronoma. What disease will this scan rule out? A. Kidney cortex B. Thyroid gland C. Pituitary gland D. Adrenal cortex

D. Adrenal cortex An aldosteronoma is an aldosterone-secreting adenoma of the adrenal cortex. An aldosteronoma is not a tumor of the kidney cortex. An aldosteronoma is not a tumor of the thyroid gland. An aldosteronoma is not a tumor of the pituitary gland.

Which hormonal deficiency reduces the growth of axillae and pubic hair in female clients? A. Growth hormone B. Antidiuretic hormone C. Thyroid-stimulating hormone D. Adrenocorticotropic hormone

D. Adrenocorticotropic hormone An adrenocorticotropic hormone deficiency causes a reduced growth of axial and pubic hair in women. A growth hormone deficiency causes decreased muscle strength and decreased bone density. An antidiuretic hormone deficiency causes excessive urine output and a low urine specific gravity. A thyroid-stimulating hormone deficiency results in hirsutism and menstrual abnormalities.

A primary healthcare provider prescribes a low-sodium, high-potassium diet for a client with Cushing syndrome. Which explanation should the nurse provide to the client about the need to follow this diet? A. The use of salt probably contributed to the disease. B. Excess weight will be gained if sodium is not limited. C. The loss of excess sodium and potassium in the urine requires less renal stimulation. D. Excessive aldosterone and cortisone cause retention of sodium and loss of potassium.

D. Excessive aldosterone and cortisone cause retention of sodium and loss of potassium. Clients with Cushing syndrome must limit their intake of salt and increase their intake of potassium. The kidneys are retaining sodium and excreting potassium. An excessive secretion of adrenocortical hormones in Cushing syndrome, not increased or high sodium intake, is the problem. Although sodium retention causes fluid retention and weight gain, the need for increased potassium also must be considered. Because of steroid therapy, excess sodium may be retained, although potassium may be excreted.

A nurse is caring for a client with Addison's disease. Which information should the nurse include in a teaching plan to encourage this client to modify dietary intake? A. Increased amounts of potassium are needed to replace renal losses. B. Increased protein is needed to heal the adrenal tissue and thus cure the disease. C. Supplemental vitamins are needed to supply energy and assist in regaining the lost weight. D. Extra salt is needed to replace the amount being lost caused by lack of sufficient aldosterone to conserve sodium.

D. Extra salt is needed to replace the amount being lost caused by lack of sufficient aldosterone to conserve sodium. Lack of mineralocorticoids (aldosterone) leads to loss of sodium ions in the urine and subsequent hyponatremia. Potassium intake is not encouraged; hyperkalemia is a problem because of insufficient mineralocorticoids. Increasing protein is needed to heal the adrenal tissue and thus cure the disease caused by idiopathic atrophy of the adrenal cortex; tissue repair of the gland is not possible. Vitamins are not directly energy-producing; nor will they help the client gain weight.

Which procedure is preferred to find out the composition of a thyroid nodule and ascertain the need for further surgical intervention in a client?' A. Mass spectrometry B. Computed tomography scans C. Glycosylated hemoglobin test D. Needle biopsy

D. Needle biopsy Needle biopsy is an ambulatory surgical procedure. A fine needle is used to aspirate the contents of thyroid nodules to study the composition and ascertain the need for further surgical interventions. Mass spectrometry is an assay in which several different hormone concentrations can be simultaneously analyzed. Computed tomography scans are useful for evaluation of ovaries, adrenal glands, and the pancreas. The average blood glucose level over 2 to 3 months is revealed by a glycosylated hemoglobin test.

What are the cardiovascular manifestations observed in a client with adrenal insufficiency? A. Fatigue B. Salt craving C. Weight loss D. Hyponatremia

D. Hyponatremia Hyponatremia is a decrease in serum sodium levels, which is the cardiovascular manifestation of adrenal insufficiency. Fatigue is a neuromuscular manifestation observed in clients with adrenal insufficiency, while salt cravings and weight loss are the abdominal manifestations observed in clients with adrenal insufficiency.

The postpartum nurse is taking the vital signs of a client who delivered a healthy newborn 4 hours ago. The nurse notes that the client's temperature is 100.2° F. What is the priority nursing action? A. Document the findings. B. Retake the temperature in 15 minutes. C. Notify the health care provider (HCP). D. Increase hydration by encouraging oral fluids.

D. Increase hydration by encouraging oral fluids.

Which statement does the nurse know is true regarding the effects of parathyroid hormone on bones for the maintenance of calcium balance? A. Increases bicarbonate and sodium excretion B. Enhances absorption of calcium and phosphorous C. Increases reabsorption of calcium and magnesium D. Increases net release of calcium into extracellular fluid

D. Increases net release of calcium into extracellular fluid Parathyroid hormone affects target tissues such as bone, kidney, and the gastrointestinal tract. The effects of parathyroid hormone on bones will be associated with the increase in the net release of calcium into extracellular fluid. Kidneys are responsible for increasing the bicarbonate and sodium excretion from the body. Action of parathyroid hormone on the gastrointestinal tract would show effects such as enhanced absorption of calcium and phosphorous. Kidneys are responsible for increased reabsorption of calcium and magnesium.

Which assessment finding in a client signifies a mild form of hypocalcemia? A. Seizures B. Hand spasms C. Severe muscle cramps D. Numbness around the mouth

D. Numbness around the mouth A numbness or tingling sensation around the mouth or in the hands and feet indicates mild-to-moderate hypocalcemia. Seizures, hand spasms, and severe muscle cramps are associated with severe hypocalcemia.

Which cells does the nurse identify as producing thyrocalcitonin hormone? A. Islet cells B. Adrenal cells C. Pituitary cells D. Parafollicular cells

D. Parafollicular cells Parafollicular cells produce thyrocalcitonin hormone. This hormone helps in the regulation of serum calcium levels. Islet cells are responsible for the production of hormones such as insulin and glucagon. Adrenal cells are responsible for the production of hormones such as cortisol and aldosterone. Pituitary cells are responsible for the production of growth hormone, prolactin, and adrenocorticotropic hormone.

What is a clinical manifestation in a client with hyposecretion of growth hormone? A. Lethargy B. Weight gain C. Decreased libido D. Reduced bone density

D. Reduced bone density Growth hormone deficiency changes tissue growth patterns resulting in increasing bone destructive activity and reduced bone density. A client becomes lethargic and gains weight due to the deficiency of thyroid stimulating hormone. Decreased libido (sexual desire) is seen due to the deficiency of gonadotropins.

A client with type 1 diabetes is admitted to the hospital for major surgery. Before surgery, the client's insulin requirements are elevated but well controlled. What insulin requirements will the nurse anticipate for this client postoperatively? A. Decrease B. Fluctuate C. Increase sharply D. Remain elevated

D. Remain elevated Emotional and physical stress may cause insulin requirements to remain elevated in the postoperative period. Insulin requirements will remain elevated rather than decrease. Fluctuating insulin requirements usually are associated with noncompliance, not surgery. A sharp increase in the client's insulin requirements may indicate sepsis, but this is not expected.

The nurse has just administered ibuprofen (Motrin) to a child with a temperature of 38.8° C (102° F). The nurse should also take which action? A. Withhold oral fluids for 8 hours. B. Sponge the child with cold water. C. Plan to administer salicylate (aspirin) in 4 hours. D. Remove excess clothing and blankets from the child.

D. Remove excess clothing and blankets from the child.

A client who is taking an oral hypoglycemic daily for type 2 diabetes develops the flu and is concerned about the need for special care. What should the nurse instruct the client to do? A. Skip the oral hypoglycemic pill, drink plenty of fluids, and stay in bed. B. Avoid food, drink clear liquids, take a daily temperature, and stay in bed. C. Eat as much as possible, increase fluid intake, and call the office again the next day. D. Take the oral hypoglycemic pill, drink warm fluids, and perform a serum glucose test before meals and at bedtime.

D. Take the oral hypoglycemic pill, drink warm fluids, and perform a serum glucose test before meals and at bedtime. Physiological stress increases gluconeogenesis, requiring continued pharmacological therapy despite an inability to eat; fluids prevent dehydration, and monitoring serum glucose levels permits early intervention if necessary. Skipping the oral hypoglycemic can precipitate hyperglycemia; serum glucose levels must be monitored. Food intake should be attempted to prevent acidosis; oral hypoglycemics should be taken, and serum glucose levels should be monitored. Telling the client to eat as much as possible, increase fluid intake, and call the office again the next day are incomplete instructions; oral hypoglycemics should be taken, and serum glucose levels should be monitored. Eating as much as possible can precipitate hyperglycemia.

All cells in the body are believed to have intracellular receptors for A. insulin. B. glucagon. C. growth hormone. D. thyroid hormone.

D. thyroid hormone. There are two types of receptors: those that are within the cell (e.g., steroid and thyroid hormone receptors) and those that are on the cell membrane (e.g., water-soluble hormone receptors). Thyroid hormone receptors are located inside the cell. Because these hormones are lipid soluble, they pass through the target cell membrane by passive diffusion and bind to receptor sites located in the cytoplasm or nucleus of the target cell.


Set pelajaran terkait

Program logic and design chapter 7

View Set

OWare- Consumer Math 10. Consumer Math Review

View Set

Chapter 10-Translation of Foreign Currency Financial Statements

View Set

Psychology unit 1 test chap 1-4 study review

View Set

Modules 1 - 4: Securing Networks Group Exam Answers

View Set

Az üzleti vállalkozás - vállalat - 2.

View Set